Василий Лурье. Философия Дионисия Ареопагита: Теория значения

Философия Дионисия Ареопагита: Теория значения*

ВАСИЛИЙ ЛУРЬЕ

.  

0. Введение 

0.1. Дионисий в качестве философа

Едва ли какое-либо исследование по средневековой философии может обойтись без упоминания Дионисия Ареопагита[1]. Тем не менее собственные философские взгляды этого автора разбираются почти исключительно с точки зрения их преемственности с теми или иными неоплатоническими учениями, особенно учением Прокла. Исследования в этом направлении имели большой успех, что, в свою очередь, привело к еще большему уходу в тень философских тем, притом что специфически христианские смыслы, представленные в сочинениях Дионисия, еще нуждаются в формулировке именно применительно к философскому содержанию мысли Ареопагита. Очевидно, что давно назрела необходимость другого подхода к Дионисию, при котором в первую очередь рассматривались бы те его философские идеи, которые выделяют его на фоне философов-неоплатоников.

При таком подходе философия Дионисия оказывается гораздо ближе к современным философским теориям, особенно связанным с философией языка и неклассическими логиками, нежели с античной философией (cp.: Zachhuber 2011). Философская история многих из этих идей обычно прослеживается вглубь веков не далее Лейбница, чье философское наследие в очень значительной степени было впервые прочитано в ХХ в.[2] Но ведь и сам Лейбниц вдохновлялся своими предшественниками. Среди них были и схоласты, и немецкие мистики, — и это как минимум два канала, через которые до Лейбница могло доходить влияние Дионисия Ареопагита (ср.: Edel 1996). Разумеется, в области собственно теологии различия между Лейбницем и Ареопагитом фундаментальны. Но в философии между ними оказывается неожиданно много общего[3].

Предварительно можно выделить по меньшей мере четыре кластера философских проблем внутри Corpus Areopagiticum:

1. «Божественные имена» — теория значения; это и главная проблема аналитической философии, хотя у Ареопагита она поставлена в связи с Богом, т. е. в необычном для аналитической философии ракурсе.

2. Логика тварного бытия, т. е. логика онтологии: у Ареопагита это специфическая система модальных логик (см.: Lourié 2013).

3. Параконсистентная логика божественного бытия[4].

4. Параконсистентная эпистемическая логика: своеобразное учение о docta ignorantia («ученом незнании») — по всей видимости, более радикальное, нежели вдохновленное Ареопагитом учение Николая Кузанского[5]. Эпистемологическое учение Дионисия неотделимо от его учения о божественном бытии, а поэтому они вместе образуют единое учение о Боге и богопознании.

Эти четыре блока, без сомнения, не исчерпывают философское содержание сочинений Ареопагита. Однако они образуют некий философский «скелет» системы его мысли. Несмотря на то что Ареопагит уделяет не меньше внимания собственно богословским и литургическим темам, его философия — важнейший raison d’être корпуса его сочинений, и ей преимущественно посвящены два трактата: «О божественных именах» и «О таинственном богословии».

0.2. Теория значения: «аналитическая философия» Дионисия

Концепция «божественных имен» имеет центральное значение для всей философской системы Ареопагита, и в этом смысле Ареопагита можно назвать аналитическим философом[6]. Точного определения аналитической философии не существует, но, во всяком случае, все согласны с тем, что это такая философия, которая очень серьезно относится к данным естественного языка. Можно сказать и иначе: это такая философия, темы которой затрагиваются в «Логико-философском трактате» Витгенштейна[7], хотя могут при этом получать совершенно иную трактовку, нежели у автора «Трактата». Ареопагит идеально подходит под это последнее «определение» аналитической философии. Заключительная фраза «Трактата» «Wovon man nicht sprechen kann, darüber muß man schweigen» («О чем невозможно говорить, о том следует молчать») (7) представляет собой одну из многочисленных альтернатив учению Ареопагита о том, как говорить о «неизреченном» (ἄρρητον), т. е. сразу и катафатическому, и апофатическому богословию, только вместе ведущих к третьему и окончательному пути «тáинственного богословия» — которое за пределом каких бы то ни было слов[8]. Витгенштейн, утверждавший в своем «Трактате», что «Gott offenbart sich nicht in der Welt» («Бог не открывает Себя в мире») (6.432), едва ли согласился бы с Ареопагитом. Тем не менее их спор вполне поместился бы в рамки аналитической философии, как ее принято понимать сегодня.

Ареопагитова теория значения никак не менее разработана, чем аналогичная теория Каппадокийцев (которая ей предшествовала и учитывалась Ареопагитом), — но, как ни странно, гораздо менее изучена[9]. Понять Ареопагита без Каппадокийцев невозможно, но и сам Ареопагит отчасти служит к толкованию Василия Великого, — и мы постараемся показать это в настоящей работе.

Историко-философское изучение Ареопагита, разумеется, должно было начаться с его неоплатонических корней[10] — это был первый и необходимый шаг. Но теперь следует продвигаться дальше. Дионисий, несмотря на свой высокопоэтический язык, в качестве философа очень и очень формален. Он не менее формален, чем Аристотель в Аналитиках, что я и надеюсь показать ниже.

1. «Гармонизация»: способ обретения значения

Такой мир, который не является миром пантеизма, т. е. не отождествляется с Богом, но в котором Бог присутствует всюду и во всем, превращается в мир знаков. В терминологии Ареопагита это мир божественных имен. Все без исключения является именем Божиим. Разумеется, это укладывается в неоплатоническую традицию. Как уже справедливо замечено, «[f]or Dionysius… as for Plotinus and Proclus, the whole of reality, all that is, is theophany, the manifestation or appearance of God» («для Дионисия… как и для Плотина и Прокла, реальность в целом, все, что существует, есть теофания, манифестация или проявление Бога») (Perl 2007, 32). Отличие Дионисия тут будет в том, что эта теофания станет основой его учения о божественных именах и именовании Бога.

Бог неименуем, потому что Он превыше любого имени, но тем не менее все многочисленные божественные имена суть Его настоящие имена. Это очевидный парадокс, и Дионисий дает свое определение божественного имени так, чтобы этот парадокс не снять, но объяснить[11]. Для этого он вводит ключевое понятие — «гармонизация» (ἐφαρμονία, ἐναρμονία и другие производные от того же корня). Соответствующее определение содержится в вводной главе трактата «О божественных именах»[12]:

Οὕτως οὖν τῇ πάντων αἰτίᾳ καὶ ὑπὲρ πάντα οὔσῃ καὶ τὸ ἀνώνυμον ἐφαρμόσει (здесь и далее в цитатах из Дионисия все выделения сделаны мной. — В. Л.) καὶ πάντα τὰ τῶν ὄντων ὀνόματα <…> Οὐ γὰρ συνοχῆς ἢ ζωῆς ἢ τελειώσεως αἰτία μόνον ἐστίν, ἵνα ἀπὸ μόνης ταύτης ἢ τῆς ἑτέρας προνοίας ἡ ὑπερώνυμος ἀγαθότης ὀνομασθείη. Πάντα δὲ ἁπλῶς καὶ ἀπεριορίστως ἐν ἑαυτῇ τὰ ὄντα προείληφε ταῖς παντελέσι τῆς μιᾶς αὐτῆς καὶ παναιτίου προνοίας ἀγαθότησι καὶ ἐκ τῶν ὄντων ἁπάντων ἐναρμονίως ὑμνεῖται καὶ ὀνομάζεται (DN 1.7, 119.10–120.8/596C–597A)[13].

Таким образом, все имена тварных существ «гармонизированы» с Вышеименной (ὑπερώνυμος) Причиной всего сущего посредством «Безымянной благости» Провидения, которым, как мы знаем из Дионисия, весь тварный мир был сотворен, продолжает поддерживаться в бытии и, в конце концов, приводится к своей полноте и завершению в обожении. Вот почему Бог одновременно и неименуемый и безымянный, и воспеваемый многоименно (πολυώνυμως, DN 1.6, 118.13/596A) и даже вообще именуемый любым именем:

<…> οἱ θεολόγοι καὶ ὡς ἀνώνυμον αὐτὴν ὑμνοῦσι καὶ ἐκ παντὸς ὀνόματος (DN 1.6, 118.2–3/596A)[14].

Поэтому божественные имена — это далеко не только имена, известные из Библии и рассматриваемые в подробностях на протяжении большинства глав трактата «О божественных именах». Напротив, имена вообще всего являются также и божественными именами. Дионисий особо уточняет, что даже имя несуществования, μὴ ὄν, не составляет исключения — оно тоже одно из имен Божиих:

Τολμήσει δὲ καὶ τοῦτο εἰπεῖν ὁ λόγος, ὅτι καὶ τὸ μὴ ὂν μετέχει τοῦ καλοῦ καὶ ἀγαθοῦ, τότε γὰρ καὶ αὐτὸ καλὸν καὶ ἀγαθόν, ὅταν ἐν θεῷ κατὰ τὴν πάντων ἀφαίρεσιν ὑπερουσίως ὑμνεῖται (DN 4.7, 152.10–12/704B)[15].

Но тут надо уточнить, что именно значит μὴ ὂν в данном контексте. Очевидно, что это не Божественное Ничто, т. е. не Сам Бог, рассматриваемый как абсолютное несуществование. А также это не тот вид несуществования, который Дионисий приписывает злу: зло — это невозможность существовать (или, на более точном логическом языке, — логический коннектив внешнего отрицания[16], который не следует путать с несуществованием как таковым, τὸ μὴ ὄν, представляющим собой отсутствие некоего тварного предмета, А, безотносительно к необходимости или возможности его существования).

Нам здесь не будут важны подробности того, как эти различные виды имен Божиих образуют основу обоих путей богопознания — катафатического и апофатического. Нам важно лишь показать, что «гармонизация» — это тот механизм, благодаря которому оба этих метода, апофатика и катафатика, оказываются работоспособными:

Καὶ ἔστιν αὖθις ἡ θειοτάτη θεοῦ γνῶσις ἡ δι’ ἀγνωσίας γινωσκομένη κατὰ τὴν ὑπὲρ νοῦν ἕνωσιν, ὅταν ὁ νοῦς τῶν ὄντων πάντων ἀποστάς, ἔπειτα καὶ ἑαυτὸν ἀφεὶς ἑνωθῇ ταῖς ὑπερφαέσιν ἀκτῖσιν ἐκεῖθεν καὶ ἐκεῖ τῷ ἀνεξερευνήτῳ βάθει τῆς σοφίας καταλαμπόμενος. Καίτοι καὶ ἐκ πάντων, ὅπερ ἔφην, αὐτὴν γνωστέον· αὕτη γάρ ἐστι κατὰ τὸ λόγιον ἡ πάντων ποιητικὴ καὶ ἀεὶ πάντα ἁρμόζουσα καὶ τῆς ἀλύτου τῶν πάντων ἐφαρμογῆς καὶ τάξεως αἰτία καὶ ἀεὶ τὰ τέλη τῶν προτέρων συνάπτουσα ταῖς ἀρχαῖς τῶν δευτέρων καὶ τὴν μίαν τοῦ παντὸς σύμπνοιαν[17] καὶ ἁρμονίαν καλλιεργοῦσα (DN 7.3, 198.12–20/872AB)[18].

«Гармонизацией» Дионисий называет непрерывно совершающийся акт «обозначения» — присваивания значения знаку, которым становится всякая тварная вещь и даже всякая несуществующая вещь. Этот знак, независимо от своего денотата, является знаком Бога. Каким же образом? В чем состоит соответствующий семантический механизм?

2. Божественные имена: нередуцируемая интенсиональность

Богословское разъяснение того, что такое божественные имена, довольно хорошо известно, поэтому здесь будет достаточно коснуться его лишь вкратце. Оно основано на учении Каппадокийцев о нетварных энергиях Божиих как основном инструменте божественного откровения. В терминологии Дионисия энергии Божии, как правило (хотя и не во всех случаях), называются διακρίσεις и πρόοδοι:

Ταύτας ἡμεῖς τὰς κοινὰς καὶ ἡνωμένας τῆς ὅλης θεότητος διακρίσεις εἴτ’ οὖν ἀγαθοπρεπεῖς προόδους[19] ἐκ τῶν ἐμφαινουσῶν αὐτὰς ἐν τοῖς λογίοις θεωνυμιῶν ὑμνῆσαι κατὰ τὸ δυνατὸν πειρασόμεθα τούτου, καθάπερ εἴρηται, προδιεγνωσμένου τὸ πᾶσαν ἀγαθουργικὴν θεωνυμίαν, ἐφ’ ᾗπερ ἂν κεῖται τῶν θεαρχικῶν ὑποστάσεων, ἐπὶ τῆς ὅλης αὐτὴν ἐκληφθῆναι θεαρχικῆς ὁλότητος ἀπαρατηρήτως (DN 2.11, 137.8–13/652A)[20].

Божественные имена относятся к Богу вообще, а не к отдельной ипостаси. Однако божественные имена, θεωνυμίαι, отнюдь не являются пассивными знаками. Напротив, они сами суть «благодейственные» (ἀγαθουργικαί), т. е. творящие благо энергии Божии. Быть «благодейственным», в терминологии Ареопагита, — это специфическое качество Самого Бога. Таким образом, божественные имена являются специфически божественными постольку, поскольку сами нетварные. «Благодейственность» обеспечивает то, что все имена Божии относятся к Богу. И именно этой «благодейственностью» обеспечивается сама познаваемость Бога (о чем много говорит Дионисий и почти исключительно этому вопросу посвящает трактат «О таинственном богословии»), которая совершается на пути преодоления как утверждения, так и отрицания.

Такова собственно богословская сторона учения Дионисия о божественных именах. Она изучена гораздо полнее, чем сторона философская, — когда божественные имена должны рассматриваться в качестве тварных термов (знаков), отсылающих к тварным денотатам. Поскольку у Дионисия разработан и этот аспект доктрины, то естественно спросить, из какой теории значения (в философском смысле этого слова) он исходит.

Эту теорию значения можно считать одной из наиболее радикальных форм семантического экстернализма, поскольку она построена на представлении о нетварных божественных энергиях: любой частный или общий терм, равно как и его отрицание, безотносительно к их прямому денотату служат обозначением Бога. Это своеобразно напоминает «новый антисубъективизм» Дональда Дэвидсона (Davidson 2001), хотя и лишь в ограниченном контексте.

Согласно Дэвидсону концептуальные схемы необходимы для осмысления нашего эмпирического опыта потому, что они являются такими же внешними по отношению к нашему сознанию. Они настолько же «объективны», насколько «объективен» сам опыт. Божественные энергии у Дионисия играют отчасти аналогичную роль этим концептуальным схемам Дэвидсона, но применительно к познанию Бога. Они точно так же не являются нашим «субъективным» вкладом в богопознание, поскольку принадлежат к реальности за пределами чего-либо человеческого. Тем не менее на Дионисия нельзя a priori перенести те представления об отсутствии внутренней связи между знаком и означаемым, которые характерны для Дэвидсона или хотя бы для современного семантического экстернализма в целом. Надо выяснить, является ли Бог именно денотатом божественных имен или чем-то другим (притом что божественные имена имеют и свои собственные, тварные денотаты).

Когда Неименуемое (τὸ ἀνώνυμον, DN 1.7, 119.10–11/596C и далее по тексту) оказывается именованным, оно остается безымянным. Оно именуется и не в смысле фрегевского Bedeutung (денотата: индивидуальных объектов или значений истинности), и не в смысле фрегевского Sinn (коннотата, или «смысла»: классов объектов) (Frege 1892). Ареопагит применяет к Богу особый вид именования, который я бы назвал «интенсиональным». Слово «интенсиональный» имело много разных значений, начиная с Лейбница, который впервые ввел его в логику и семантику, заимствовав из средневековой физики. Мы определим его сейчас наиболее широким способом из возможных, а именно: интенсиональное значение — это все те компоненты значения, которые не являются экстенсиональными, т. е. не являются ни индивидуальными объектами (тварного мира), ни значениями истинности. Одно из преимуществ такого определения интенсиональности — это, между прочим, его сходство с пониманием интенсиональности в зрелой философии Лейбница (см.: Lourié 2012), от которого этот термин и пришел в современную философию.

Это не означает, однако, что мы не будем использовать и обычное понятие интенсиональности, как оно было введено Рудольфом Карнапом (Carnap 1947) в соответствии с линией мышления Фреге[21], где интенсиональность — это функция от возможностей к экстенсионалам. В рамках такого подхода интенсиональная семантика строится как редукция интенсионала к экстенсионалу (часто так и называемая «экстенсионалистской редукцией» (Shani 2005)).

Согласно самому Карнапу, интенсиональная компонента значения должна быть редуцируемой к экстенсиональной компоненте — по крайней мере, в языке науки. Он выразил эту свою надежду в так называемом «Тезисе экстенсиональности»: «Универсальный язык науки должен быть экстенсиональным». Карнап предложил этот тезис «лишь в качестве предположения», но считал его «очень правдоподобным» (Carnap 2001, 245–247). Однако этот тезис оказался достаточно проблематичен даже в естественных науках[22], а в богословии Дионисия он оказался бы точной противоположностью тому, что есть на самом деле: «универсальный язык богословия» у Дионисия нередуцируемо интенсиональный[23].

Когда божественные энергии творят порядок для всех объектов всего мира, они являются законными наследницами платоновских идей и, в частности, классическим примером «интенсиональных объектов». Поэтому интенсиональность в семантике Ареопагита и в его соответствующем языке нередуцируема.

Бог является интенсионалом божественных имен в том смысле, что Он «имплицируется» в них. Конечно, такая импликация имеет специфический механизм, поскольку Бог считается реально присутствующим в именах, — но для логической семантики не важно, является ли это онтологическое утверждение истинным. Важно лишь то, что оно принимается в качестве истинного. Само это «богословие реального присутствия» Бога в Его символе будет разрабатываться гораздо позже — в дискуссиях VIII, IX и XI вв., посвященных святым иконам (cр.: Lourié 2000; Lourié 2006). Но, повторим, сейчас нас интересует не богословие как таковое, а только его логическое оформление. Мы подходим к богословию Дионисия, как мог бы выразиться Куайн, «с логической точки зрения».

Едва ли существует какая-либо современная логико-философская система, которую можно было бы взять в готовом виде для объяснения философского смысла того, что говорит Дионисий. Тем не менее учение Дионисия о Боге как интенсионале божественных имен перекликается с рядом современных философско-логических идей, и поэтому современной аналитической философии есть что сказать полезного для нашего понимания Дионисия.

3. Употребление превосходной степени как маркер центрированного в Боге логического универсума

В классической одномировой семантике интерпретация Дионисиевых божественных имен встречает непреодолимые трудности. В ней некий конкретный бог должен был бы оказываться денотатом некоего конкретного имени божия. Интенсионал должен был бы стать той функцией, которая отбирает одного нужного бога из множества, т. е. из соответствующего класса (класса богов). Но у Дионисия такого класса нет: во всяком случае, нет такого класса, хотя бы чисто теоретического, в котором существует множество богов (более одного).

В многомировой семантике интенсиональность — это функция от миров к референтам. Если бы она была применима к нашему случаю, то она позволяла бы выбрать из множества возможных миров тот, в котором Бог только один, и это именно Бог Дионисия, к которому и относятся все божественные имена. Но даже если это множество возможных миров представлять себе в духе Крипке, т. е. считая, что лишь один из них — наш актуальный и реально существующий мир, а все остальные миры суть не более чем интеллектуальные конструкты (см.: Kripke 1963 (переиздавалось много раз))[24], этого все равно не будет достаточно для понимания Дионисия. Проблема в том, что внутри lUnivers dionysien[25] не имеется даже эпистемической возможности различных богов или различных миров с другими единственными богами. Внутри этого универсума мы имеем только те эпистемические возможности, которые предоставляются божественными энергиями, т. е. реальным присутствием Самого Бога. Таков закон познания как обожения — единственного рода познания, которым располагает истинное богословие согласно Дионисию. Следовательно, для Дионисия сама возможность выбора истинного Бога между разными возможностями (из которых все остальные суть возможности выбрать ложного бога) находится уже вне его логического универсума. Его собственная личность — личность афинского философа, обращенного в христианство апостолом Павлом, — является символом осуществления такого выбора универсума.

На собственном языке Дионисия результат выбора особого универсума обозначается с помощью использования превосходной степени — приставки «сверх-», ὑπερ-. Прежде всего, это касается у Дионисия его противопоставления Бога христиан всем прочим богам. Поэтому для истинного Бога не подходит слово «бог». Дионисий использует вместо него слово «Сверхбог» — ὁ ὑπέρθεος (DN 2.10, 135.4/648D)[26]. Так обозначается результат выбора между разными концепциями бога. Однако важно, что соответствующий акт выбора происходит не внутри Дионисиева логического универсума, а еще заранее, т. е. вне его, и соответствующая эпистемическая процедура еще не является тем познанием-обожением, которое дается божественными именами. Это лишь предварительное условие, которое делает возможным обсуждение божественных имен.

Таким образом, вне логического универсума Дионисия терм «бог» функционирует как индексикал: имеется множество эпистемических возможностей для его понимания, хотя лишь одна из них истинная — в качестве «Сверхбога». Индексикалами называются термы и выражения, референция которых меняется от контекста к контексту (типичный пример — местоимения, например: «я», «он» и т. д.). Терм «бог», взятый вне Дионисиева универсума, ведет себя как индексикал: значения истинности любого выражения, содержащего этот терм, будут зависеть от контекста выбранной нами богословской системы (т. е. от нашего понимания, кто или что такое «бог»). Однако внутри Дионисиева логического универсума имеется только одна эпистемическая возможность, и она реализована.

Теория божественных имен не относится к «богу» как к индексикалу. Она лишь о едином и единственном Боге. Терм «бог» оказывается своего рода прономиналом в такой логической конструкции, в которую уже подставили специфические имена, и любое из них при этом подставляется в качестве конкретного имени в этот прономинальный терм.

При входе в логический универсум Дионисия мы должны вынести за скобки индексикальность терма «бог». Говоря более формально, мы можем использовать удобный логический аппарат двумерной семантики (см.: Chalmers 2010, 541–568 (Appendix: Two-Dimensional Semantics))[27]. В терминологии двумерной семантики мы можем сказать, что Дионисий внутри своего собственного логического универсума работает с единственным сценарием идентификации Бога. Это означает, что его универсум центрирован на его Боге (или, точнее, Сверхбоге). Так Бог Дионисия перестает быть индексикалом — поскольку соответствующий ему логический универсум центрирован.

Употребляемые Дионисием превосходные степени всегда указывают на единственность истинного Бога, и таким образом они становятся маркерами центрированности его логического универсума.

С богословской точки зрения эта центрированность означает точку прекращения эпистемической активности, направляемой человеческим умом, и начала эпистемической активности, управляемой божественными энергиями. Тем не менее в обоих случаях человеческая деятельность и деятельность божественная работают вместе. Отличие лишь в том, что лидирует[28]. Логический универсум становится ὑπέρθεος-центрированным тогда, когда человеческая активность сменяется на пассивность (правда, такую пассивность будет точнее назвать «активной пассивностью») — чтобы уступить рабочее место активности Бога.

Разумеется, двумерная семантика не является единственно возможным способом формальной интерпретации превосходных степеней у Дионисия, но, во всяком случае, это полезный инструмент, чтобы представить положение Дионисиева логического универсума в качестве «встроенного» в более широкую интеллектуальную конструкцию, в которой христианский Бог еще не определен в качестве единственного истинного. Дионисий начинает свое богословие с той точки, в которой эта конструкция рушится, но он никогда не забывает о том, что она была в его прошлом[29].

4. Именование Неименуемого: интенсиональная десигнация

До сих пор мы рассматривали ситуацию перед входом в логический универсум Дионисия, т. е. вне этого универсума. Поэтому мы говорили о том, как индексикал «бог» обретает свой единственный, по Дионисию, денотат. Это вполне классическая семантика: реальный Бог считается денотатом некоего терма. Однако как только мы вступаем в Дионисиев универсум, все резко меняется: Бог перестает быть денотатом. Вместо одного имени Он обретает множество имен, но при этом остается неименуемым, когда Его именуют. Но каждое из этих многих имен имеет свой собственный денотат. Среди денотатов имен, помимо индивидуальных вещей и ситуаций, также и отсутствие каждой из них (их отрицание), а также отсутствие (отрицание) их всех — μὴ ὄν; последнее означает, что, по Дионисию, «дырка» в бытии, оставшаяся после исчезновения некоего объекта, не менее реальна, чем сам объект[30].

Обычно индексикал в центрированном возможном мире приобретает единственный денотат, который, в свою очередь, может иметь свое имя собственное. Вместо этого в универсуме Дионисия Бог приобретает множество денотатов и множество имен. Теоретически говоря, эта множественность имен может быть интерпретирована как множественность дескрипций через предикаты (такие как предикат «быть 〚ni〛», где 〚ni〛— это денотат божественного имени ni), и тогда подобная многоименность не будет представлять собой самостоятельной проблемы. Но тут возникает другая проблема: Богу невозможно приписывать тварные предикаты[31]. В классической семантике из этого положения нет никакого выхода. Получается, что божественные имена в любом случае «неклассические», безотносительно того, будем ли мы их рассматривать как дескрипции или как имена, несводимые к дескрипциям (тут оба этих подхода возможны, но оба потребуют какой-то неклассической трактовки).

Концепция жестких десигнаторов Крипке тут может быть полезна для сравнения (и для противопоставления). Понятие жесткого десигнатора подразумевает, что существуют некие термы (и, в частности, к ним относятся собственные имена), которые обозначают один и тот же объект в любом из возможных миров, т. е. эти термы обладают поведением, абсолютно независимым от контекста (Kripke 1980)[32].

Между прочим, интерпретация собственных имен разумных творений как жестких десигнаторов является необходимой, хотя и скорее имплицитной составляющей патристической антропологии в части опровержения учения о метемпсихозе: имя относится к каждому разумному существу, ангельскому или человеческому, которое идентично само себе в пределах своей вечной жизни. Идентичность человеческого существа самому себе на протяжении земной жизни, далее после разделения души и тела и затем воскресения никак не может быть передана внешней дескрипцией того или иного из (столь различных!) актуальных состояний. Насколько мне известно, нигде в патристической литературе не ведется обсуждение индивидуальности и имен собственных в таком — вечном — контексте посредством дескрипций. В то же время классический для патристики дескрипционистский подход к трактовке имени собственного обычно соотносится с актуальным состоянием носителя имени (см. ниже, раздел 13). Поэтому концепция жестких десигнаторов Крипке может быть исключительно полезной для понимания патристических представлений о собственных именах тварных существ[33]. Судя по косвенным признакам, это и вполне в духе Ареопагита, но прямо об именах тварных существ он не рассуждает.

Но с именами божественными связаны свои особые проблемы. Во-первых, вся иудео-христианская традиция не расположена к тому, чтобы давать Богу какие-либо конкретные имена. Как заметил по этому поводу Кристофер Стид, «…we might suppose that it is normally the function of a personal name to pick out an individual within a class of similar beings; in this case, to apply a personal name to God would be to suggest that he is not unique» («…мы могли бы предположить то, что обычно функцией личного имени является выбор некоего индивидуума внутри класса подобных существ; но в этом случае употребить собственное имя Бога означало бы дать понять, что он не является уникальным») (Stead 1988, особ. 317)[34].

У Дионисия вместо взаимнооднозначных отношений между именем собственным и его носителем, разумным существом, во всех возможных мирах мы имеем отношение единственного Бога к множеству имен в единственном центрированном (в смысле двумерной семантики) мире. Взаимнооднозначные отношения существуют, однако, между каждым из этих имен и их собственными денотатами (экстенсионалами; или фрегевскими классами, т. е. интенсионалами, но редуцируемыми к экстенсионалам, что для нас эквивалентно). В отличие от собственных имен тварных существ, такие имена не могут относиться к Богу как своему денотату: место денотата у них занято другим. Поэтому было бы разумно принять, что к Богу они относятся через интенсиональную составляющую своих значений, причем в той ее части, которая нередуцируема к экстенсионалу. Это явление, которое мы сейчас обозначим «интенсиональной десигнацией», не является чем-то специфически богословским, а является лишь формальным логическим механизмом, заимствованным Дионисием из поэзии (особенно из поэтической школы Нонна Панополитанского[35]). Такова в главных чертах структура поэтических тропов (метафоры и метонимии). Формальное различие между божественными именами Дионисия и поэтическими тропами будет лежать главным образом в онтологических презумпциях. Последнее тоже очень важно понимать, чтобы не перестать читать сочинения Дионисия как богословие и не начать их читать как простую поэзию. Но без поэзии Дионисий не обходится.

5. Именование неименуемого как поэзия

Понимание логической структуры поэтического тропа есть необходимое, хотя и не достаточное условие понимания Дионисиева учения о божественных именах. Однако для современной логики этот вопрос достаточно сложен сам по себе. Научного консенсуса в этой области нет, а есть, напротив, довольно жаркая полемика. Нам придется сформулировать свой собственный взгляд, который будет ближе всего к логической теории метафоры Хинтикки и Санду (1994), но отличаться от всех ранее высказанных взглядов учетом параконсистентной логики.

Не углубляясь в настоящей работе в подробности параконсистентной логики у Дионисия, мы все же должны будем кратко описать божественные имена как параконсистентные логические объекты. Имена Божии различны, но все они одинаково приложимы к Богу. В качестве денотатов они не являются необходимым образом взаимоисключающими, но это возможно. Таким образом, в отношении к Богу они связаны инклюзивной дизъюнкцией, или логическим суммированием (∨). Все вместе они формируют некое счетное множество, т. е. множество, содержащее количество элементов, определяемое натуральным числом, а также пустое множество (поскольку μὴ ὄν также является именем Бога). Каждый элемент этого множества входит в множество божественных имен вместе со своим отрицанием, потому что отрицание любого божественного имени — это также божественное имя. Таким образом, если мы обозначим любое «позитивное» (не содержащее отрицания) божественное имя как ni, и учитывая, что отрицание пустого множества есть также пустое множество, мы можем представить все элементы Дионисиева множества божественных имен в виде следующей конъюнкции:

(1)  

Эта конъюнкция является параконсистентной на двух уровнях. Сама по себе непосредственно она является параконсистентной в том смысле, что она основана на контрадикторном противоречии (a ∧ ¬a). На более глубоком уровне параконсистентность появляется (хотя и не всегда) внутри инклюзивной дизъюнкции, поскольку инклюзивная дизъюнкция (в отличие от эксклюзивной дизъюнкции ⊕ «точно только один из двух») допускает сосуществование таких имен, денотаты которых (или эквивалентные денотатам интенсионалы) являются несовместимыми в классической логике, так как между ними возможно контрарное противоречие (см. ниже, раздел 8).

Как контрарные, так и контрадикторные противоречия присущи поэтическому языку — и в светской, и в христианской поэзии. При этом контрарные или контрадикторные противоречия на уровне поэтических тропов совсем не обязательно подразумевают наличие этих же противоречий в реальности. Так, например, в знаменитом «Акафисте» (VI в.)[36] рефрен «Радуйся, Невесто неневестная» не подразумевает параконсистентности в реальном мире, поскольку нет ничего параконсистентного в том, чтобы быть одновременно невестой в одном смысле, но не быть невестой в другом. Однако если мы попробуем пересказать смысл этого текста без использования поэтического тропа, применив вполне адекватную дескрипцию, вся поэтическая сила стиха пропадет. Это значит, что самой метафоре параконсистентность присуща. Параконсистентность поэтического тропа нередуцируема даже тогда, когда он не претендует на выявление какой-либо параконсистентности в реальной жизни. Но часто он и претендует. Например, в чувствах: «Odi et amo…» — «Ненавижу и люблю…» (Катулл); или в субъективной, но реальности: «Мне и спится и не спится. Филин снится и не снится…» (Соснора).

6. Божественные имена в сравнении с метафорой и метонимией

Мы не имеем здесь возможности проследить подробно историю начавшихся в 1970-е гг. и до сих пор весьма острых споров между логиками относительно природы метафоры[37]. Будет достаточно сказать, что я полностью поддерживаю критику Дэвидсоном всех попыток редуцировать смысл метафоры к дескрипции (т. е. всех вариантов классического подхода, восходящего к «Поэтике» Аристотеля)[38], но при этом я не могу согласиться с его радикальным исключением метафоры из сферы семантики и переводом ее в сферу прагматики (ср.: Davidson 1991b)[39]. Mutatis mutandis, то же самое следует повторить о метонимии. Мое собственное понимание метафоры и метонимии следует в общих чертах подходу Хинтикки и Санду (Hintikka, Sandu 1994)[40], но далее мною делается еще один шаг: я утверждаю, что метафора и метонимия суть параконсистентные логические объекты.

Согласно Хинтикке и Санду, метафора и не редуцируется к сравнению, основанному на сходстве свойств, но и не может быть понята вообще без обращения к сравнению. Подобно сравнению, метафора указывает на некое сходство свойств (предикатов). Аналогично метонимия указывает на некую релевантную смежность разных объектов (добавим, что если перевести это рассуждение в категориальный аппарат «ментальных пространств», то тогда вообще все тропы будут сведены к метонимии)[41]. Однако и метафора, и метонимия идут дальше, чем просто сравнение: они устанавливают то, что Хинтикка и Санду называют «meaning lines» («смысловые линии») между носителями свойств (субъектами предикатов). Смысловые линии основаны на релевантных свойствах (предикатах). Они проводятся между разными возможными мирами и соединяют характеристические множества индивидуумов в каждом из возможных миров, соответствующих релевантному предикату, — однако без идентификации самих этих индивидуумов. В противном случае смысловые линии превратились бы в линии межмировой идентификации индивидуумов, т. е. в «межмировые линии» в смысле Дэвида Каплана.

Хинтикка и Санду интерпретируют свои «мировые линии» в качестве устанавливающих лишь некий вид межмировой идентичности, однако не экзистенциальную идентичность. Смысловые линии не суть межмировые линии, которые, в отличие от первых, основаны не на сходстве и не на смежности, а на континуальности, поскольку именно континуальность является критерием межмировой идентификации индивидуумов[42].

До этого места я полностью соглашаюсь с Хинтиккой и Санду, но я не могу согласиться с тем фактом, что они на этом и останавливаются. У них получилось убедительное описание анатомии поэтического тропа, но не его физиологии. Они не объясняют, как и почему поэтический троп «работает», т. е. почему он таким особым образом воздействует на мышление. Тем не менее в качестве посмертной аутопсии поэтического тропа, их анализ чрезвычайно важен.

На мой взгляд, физиология поэтического тропа состоит в том, что его смысловые линии начинают работать (жить) лишь тогда, когда их помещают в параконсистентные рамки. Именно параконсистентность отличает сам поэтический троп от того, что в нем обнаруживается после вскрытия методами классической логики.

В отличие от поэтического тропа, божественные имена относятся к идентификации индивидуумов. Неважно, что Бог при именовании остается неименованным: божественные имена все равно суть истинные имена Бога, причем в том смысле, что каждое из них есть Бог — ровно в том смысле, что неименуемый Бог именуется. Необходимое для идентификации индивидуумов отношение континуальности обеспечивается божественными энергиями (которые Дионисий называет по-разному: ἀγαθουργίαι, πρόοδοι, διακρίσεις и т. д.), присутствующими в каждой тварной вещи в качестве ее нетварного логоса.

Конечно, наиболее очевидные параллели к такому пониманию божественных имен можно найти в магии. Однако магия подразумевает определенную прагматику, особенно ту, что относится к магической силе над сверхестественными явлениями. И, самое главное, сама онтология божественного присутствия в божественных именах может теоретически быть очень разной. В магии она одна, в патристическом учении о божественных логосах в твари (не только у Дионисия) — другая. Поэтому — как, впрочем, хорошо известно и без меня — не существует серьезных оснований для магической интерпретации Дионисия. Но в данной работе мы не обсуждаем ни онтологию, ни прагматику, а обсуждаем только семантику. А в этом частном аспекте сходство с магией есть, и оно вполне существенное.

7. Именуя Неименуемое: параконсистентный эссенциализм

Поскольку нам приходится обсуждать межмировую идентичность, мы не сможем совсем не касаться Дионисиевой онтологии. Мы постараемся лишь как можно меньше уклоняться от центрального для нас сейчас вопроса: проблемы идентичности между Богом и не-Богом.

Как мы постарались показать в другом месте, Дионисиев универсум являет собой логически элегантную картину иерархий, в которой разные онтологические уровни соответствуют разным уровням обожения (Golitzin 1994, 86–91 и везде; cр.: Lourié 2013). При отсутствии какого бы то ни было обожения невозможно никакое бытие. Всякое бытие обожено, но в разной мере. Нулевой уровень обожения — это обладание нетварным божественным логосом, который имеется в каждом тварном бытии. Каждый следующий уровень в иерархии соответствует дополнительным логосам. Бог же превыше всех иерархий и является «конечной целью» всех иерархий (иерархии не только статичны, но и динамичны, а поэтому имеют «цель»).

Логический формализм, адекватный подобной онтологии, может быть похож на тот, что был предложен В. Л. Васюковым для формализации мейнонговской концепции Außersein (ср.: Васюков 2004; Васюков 2005а)[43]. В таких логиках не бывает просто бытия и небытия, а бывает несколько разных видов того и другого.

Поскольку в данном случае нас не волнуют подробности Дионисиевой онтологии, мы позволим себе не обсуждать иерархии сами по себе, а ограничиться лишь их фундаментальной логической структурой.

Фундаментальная идея, лежащая в основе понимания Дионисием тварного бытия, двояка и параконсистентна. С одной стороны, тварное бытие резко отличается от божественного бытия — настолько, что если тварные существа «существуют», то Бог — не существует, а также и напротив: если существует Бог, то не существует ничего тварного. Именно в последнем смысле Дионисий называет Бога «ничто», и это «ничто» не следует путать с другим «ничто», которое является одним из божественных имен, имеющих тварный денотат[44]. С другой же стороны, все, что существует, существует лишь благодаря своему обожению через божественные логосы, и таким образом оно причаствует божественному бытию. Божественное и тварное бытие остаются несмешанными, но тварное невозможно без божественного. Это концепция бытия как причастия Богу разительно отличается от всех видов пантеизма именно своей параконсистентностью: всякое утверждение о божественности твари верно лишь в том случае, если его брать одновременно с утверждением о небытии твари (или небытии Бога).

Несмотря на то что такой параконсистентный подход был отчасти знаком и западной философской и богословской мысли (не в последнюю очередь, вдохновлявшейся Ареопагитом: тут можно назвать Мейстера Экхарта, Таулера, Сузо и даже, вероятно, Якоба Бёме), его часто путают с его консистентной альтернативой, которая и стала мейнстримным подходом в схоластике (и в схоластическом толковании Ареопагита — в комментарии к нему Альберта Великого, а там и до новейшего времени включительно). Поэтому, вероятно, тут будет уместно подчеркнуть, что, по Дионисию, Бог не есть совершенное существо и нечто вроде предельного случая («совершенства») каких-либо тварных свойств. Бог не является «совершенным бытием», потому что Он вообще не является бытием — ни в каком из смыслов этого слова, приложимых к тварным существам[45]. Те иерархии, чьей конечной целью является Бог, суть иерархии различных уровней обожения, а не иерархии тварных существ, взятых именно в своем качестве тварности. Поэтому в рамках Дионисиева подхода и невозможно предложить общую онтологию для Бога и тварных вещей, и даже назначение Богу места «предельного случая» («совершенства») тварных предикатов тут не поможет. Бог вообще не является субъектом тварных предикатов, а тварные предикаты не являются результатом «разбавления» предикатов божественных. Бог Дионисия — это совершенно точно не Бог онтологических доказательств бытия Божия от Ансельма Кентерберийского до Курта Гёделя.

Тем не менее обычная логика предикатов к Богу применима, но только необычным — неклассическим — способом: тут можно сформулировать принцип, аналогичный «принципу соответствия» Нильса Бора (принципу применимости ньютоновской физики в квантовой механике: так, к квантовым объектам применимы законы классической механики и для частиц, и для волн, только в классической механике они не могут относиться к одному и тому же объекту, а в квантовой относятся, что регулируется соотношением неопределенностей Гейзенберга). Ареопагит также формулирует свой собственный «принцип соответствия», которым и становятся его катафатическое и апофатическое богословие, взятые вместе.

Оборотной стороной столь специфической онтологии для божественного бытия оказывается многослойная структура онтологии для творений. В первом (и для нас достаточном) приближении она может быть представлена с помощью аппарата алетической модальной логики как структура двуслойная. Кратко перечислим основные аксиомы этой логики для нашего случая.

Для каждого индивидуального тварного объекта и для каждого класса тварных объектов имеются специфические божественные логосы. Логосы отличаются друг от друга параконсистентным образом, т. е. отличаются, не отличаясь: Бог разделяется на энергии-логосы, но так, что остается неразделимым. Без божественных логосов все тварное является несуществующим (во всех смыслах этого слова) и даже невозможным. Но зато со своими логосами они становятся не просто возможными, но необходимо-сущими, однако не все, что является необходимо-сущим, существует в реальности (хотя все — возможно). Реальность не содержит всего того, что необходимым образом должно в ней быть. Необходимо должно быть (и оно будет) совершенное обожение всей твари (1 Кор. 15:28), но его в наличии нет.

Поэтому не выполняется так называемая модальная аксиома («все, что необходимо, имеет место в действительности»):

(M)  □A A

Многие вещи и события, на которые есть воля Божия, будучи необходимыми (как и все, на что есть воля Божия), не имеют места в действительности. Некоторые даже не будут иметь места в ней никогда (поскольку Ареопагит также придерживается учения о вечности мучений, а чья бы то ни было вечная погибель противоречит воле Божией, т. е. необходимости).

Изначальное несовершенство тварного мира — который ведь не был сотворен с самого начала в совершенстве обожения, — а также потом еще и наличие зла не допускают воле Божией быть исполненной во всех случаях в каждый данный момент[46]. Поэтому вместо аксиомы (М) действует более слабая аксиома (D), которую часто называют деонтической, так как она фундаментальна для деонтической модальной логики («если А обязательно, то А допустимо», что эквивалентно алетическому «что необходимо, то может иметь место»):

(D)  □A → ◊A

Ситуация в онтологии очень похожа на ту ситуацию, которая типична в деонтике: если что-то должно быть сделано, то есть возможность, что оно будет сделано (равно как и не сделано).

Зато так называемое necessitation rule (NR) — правило, обратное аксиоме (М), — выполняется даже строже, чем в «обычной» модальной логике:

(NR)  A → □A

В данном случае, оно подразумевает, что если нечто имеет место, то это означает, что оно должно иметь место необходимым образом. Если трава зеленая, то она зеленая необходимым образом, — тут это рассуждение не является логической ошибкой, в качестве которой его обычно приводят.

Будучи приложенным к упрощенной (до двуслойности — посредством «вынесения за скобки» подробностей иерархических структур) онтологии Ареопагита, NR означает, что всякое существующее тварное бытие (индивидуум, класс или даже ситуация) необходимым образом имеет некую причастность к божественному бытию. Никакое однослойное (без божественных логосов) существование тварных вещей невозможно, а поэтому и не может быть необходимым.

Поскольку именно божественные логосы обеспечивают идентичность тварных вещей самим себе (в том числе межмировую идентичность), то к ним можно отнести современный логический термин «essential property» («сущностное свойство») — то свойство, по наличию которого можно судить об идентичности априорно не идентичных объектов[47]. Но это и не «сущностное свойство» в смысле аристотелевой «сущности» (независимо от того, как бы мы захотели понимать эту концепцию Аристотеля, которую всегда понимали по-разному), и не сущностное свойство в смысле обычных концепций современного эссенциализма. В системе Дионисия невозможны такие объекты, которые не разделяют с другими объектами свойства иметь божественные логосы, равно как и такие объекты, которые разделяют с другими объектами свойство иметь именно свой конкретный божественный логос (хотя, впрочем, логосы отличаются друг от друга только параконсистентно, т. е. отличаются, не отличаясь)… Видно, что почва для сопоставления с современными эссенциалистскими концепциями имеется, но это далеко не тождество концепций.

Божественные логосы, поскольку они божественные, устанавливают межмировую идентичность между тварными вещами и Богом, но не индивидуализируют свои тварные объекты. В то же время божественные логосы, поскольку они различаются по «вверенным им» созданиям (т. е. поскольку они являются логосами в тварных вещах), индивидуализируют свои тварные объекты, но не устанавливают их межмировой идентичности с Богом. Таким образом, если мы рассматриваем божественные логосы как свойства тварей, то они не являются такими свойствами, которые сразу и устанавливают межмировую идентичность, и индивидуализируют «свои» объекты в ряду всех прочих тварных объектов, как это требовалось бы в современном эссенциализме для «сущностных свойств». Вместо этого они или устанавливают межмировую идентичность, или индивидуализируют свои объекты, но при этом обе функции находятся в параконсистентной конъюнкции.

Другими словами, взятый в качестве предиката (в весьма специфическом смысле этого слова) тварного существа, божественный логос должен пониматься как параконсистентная конъюнкция двух несовместимых свойств.

8. Именуя Неименуемое: Принцип соответствия с классической логикой предикатов

Современный читатель «О божественных именах», если он читает невнимательно, может подумать, будто Дионисий нам объясняет, что и кто есть Бог при помощи классической логики предикатов. Впрочем, так читали Ареопагита и многие внимательные читатели — например, Альберт Великий. Действительно, Дионисий все время утверждает, что Бог «есть» то-то и то-то, а затем, правда, утверждает, что Бог «есть» еще что-то другое, и еще что-то, и еще что-то… Все перечисляемое предполагается известным читателю, так как все это какие-то земные вещи. Таким образом, читатель становится способным понять, что и кто есть Бог. Но не все так просто. Здесь всего лишь имитация — а, точнее сказать, расширение — классической логики предикатов. Соответствие с логикой предикатов имеется, но лишь до известной степени.

Бог «есть» то-то и то-то, но в то же время Он «не есть» именно это то-то и то-то. Этот факт не влияет на внутреннюю семантику соответствующих утверждений (с «есть» или «не есть»). Взятые отдельно, они не параконсистентные, а вполне классические. Параконсистентность заключается только в одновременном их отнесении к одному и тому же субъекту. Именно поэтому предложения, построенные по модели «божественность есть (нечто)», которыми так изобилует книга «О божественных именах», не могут быть истолкованы в рамках классической логики. Иными словами, мы не сможем, зная значения истинности указанных предложений, фиксировать референцию терма «божественность» (или его синонимов). Однако так обстоит дело не с одним только Богом. Та же самая проблема возникает и при интерпретации предложений, содержащих поэтические тропы (метафоры или метонимии).

Хинтикка и Санду настаивают на том, что не бывает никакой особой «метафорической истины»: для предложений, содержащих поэтические тропы, понятие истины то же самое, что и для всех остальных предложений. Особым является лишь «a special sense of meaning (special kind of interpretation)» («особый смысл значения (особый вид интерпретации)») (Hintikka, Sandu 1994, 172)[48]. Этим особым видом интерпретации является смысловая линия, т. е. нестандартная (необычная) функция значения, существующая отдельно от интенсионала во фрегевском смысле[49] и даже, напротив, по контрасту с фрегевским интенсионалом обоих термов, связанных через поэтический троп. Такая ситуация не является особо прозрачной для референции в каком бы то ни было фрегевском смысле, и поэтому есть повод усомниться в ее фрегевости.

Во всяком случае, очевидно, что объяснения Ареопагита «работают» каким-то неклассическим способом. Или, говоря более точно, классический способ логического объяснения в патристике — способ Ареопагита — оказывается не очень-то классическим для современной логики. Теперь самое время спросить, каково значение всех Ареопагитовых пропозиций типа «божественность есть…» (свет, тьма, нечто, ничто и т. д.), т. е.

(2)  αi : божественность (θεότης) «есть» ni

и его отрицания

(3)  ¬αi : божественность (θεότης) «не есть» ni ,

где αi — это предложение (формула), ni — божественное имя (терм), а i — натуральное число (i ∈ ℕ+)[50].

Все множество {αi} является множеством формул, имеющих одинаковое значение истинности («истинно»), равно как все множество {¬αi} является множеством формул, также имеющих одинаковое значение истинности (тоже «истинно»). То, что формулы обоих множеств имеют одновременно одинаковые значения истинности, является выражением параконсистентности всей системы божественных имен. Но сейчас мы должны будем оценить один параметр этой системы — ее (не)фрегевость, а поэтому будем говорить не о параконсистентности, а о возможности узнать значение терма «божественность», если мы знаем значения остальных термов и значения истинности предложений из обоих множеств {αi} и {¬αi}.

Если бы у нас была фрегевская семантика, то формулы (2) и (3), имеющие одинаковые значения истинности, оказались бы логически эквивалентными:

(4)  αkαj

(5)  ¬αk ↔ ¬αj

при любых k, j ∈ ℕ+.

По Фреге, все они имеют один и тот же денотат, а именно — их значение истинности. Поэтому и формулы αi, и формулы ¬αi означают один и тот же денотат — значение истинности «истинно»[51]. Поэтому они бесполезны для какого бы то ни было последующего логического исследования терма «божественность». Очевидно, однако, что позиция Дионисия — а заодно и вообще всей патристической традиции не только после, но и прежде него — иная. По Дионисию, множества формул {αi} и {¬αi} выражают некую нетривиальную истину. Значит, референцией (денотатом) соответствующих предложений является нечто другое, нежели их значения истинности. У этих предложений разные денотаты, хотя значения истинности одинаковые. Это возможно лишь в том случае, если так называемая Фрегевская аксиома (FA) не исполняется. Термин «Фрегевская аксиома» был введен Романом Сушко (Suszko 1975; cр. также: Golińska-Pilarek, Huuskonen 2005), который эксплицировал это основное допущение фрегевской семантики:

(FA)  (pq) → (pq)

Если FA не исполняется, то логическая эквивалентность формул (предложений) p и q не влечет за собой идентичности их референций (денотатов). Это, в свою очередь, означает, что не выполняется принцип ко-референциальности эквивалентных формул, CE (предложения имеют одни и те же денотаты, если они логически выводимы друг из друга) (cр.: Горбатов 2011) (⊦ означает «логически выводимо»):

(CE)  Если AB и B A, то A B

В тех системах логики, где FA и CE не выполняются, логически эквивалентные предложения не являются необходимым образом ко-референциальными, но при этом ко-референциальные предложения необходимым образом эквивалентны. Таким образом, имеет место аксиома нефрегевости (NFA) (идентичность денотатов подразумевает эквивалентность):

(NFA)  (pq) → (pq)

Тогда денотатами предложений являются ситуации, которые в них описаны (а не их значения истинности). Семантики, основанные на этом принципе, называются «ситуационными»[52].

Сушко ввел коннектив идентичности ≡ в качестве неистинностного функционального коннектива, отличного от логической эквивалентности (↔). Предложения эквиваленты только в том случае, если они имеют одинаковые денотаты, т. е., согласно основному постулату ситуационной семантики, если и только если описанные в них ситуации идентичны. Возвращаясь к Дионисию, мы видим, что его предложения {αi} и {¬αi}, хотя и являются эквивалентными согласно (4) и (5), не являются идентичными в смысле Сушко (и какой бы то ни было вообще ситуационной семантики):

(6)  αkαj

(7)  ¬αk ≢ ¬αj

Поэтому они не тривиальны и не тавтологичны, а, следовательно, ничто не мешает им иметь значение, важное для уразумения того, что и кто есть Бог.

9. Божественные имена: онтологизация тропов

Нефрегевская семантика позволяет нам продолжить изучение Дионисиевых божественных имен в сравнении с поэтическими тропами. «Негативная» аргументация Куайна и Патнема, которая в свое время была сформулирована, чтобы показать «мутность» («opacity») референции в теориях, основанных на представлениях Фреге, будет иметь положительное значение в контексте нефрегевской семантики.

Куайн привел примеры ситуаций, где, вопреки Фреге, условия истинности предложений в целом не позволяли определить референцию[53]. Патнем в своей знаменитой теореме показал, «…that there are always infinitely many different interpretations of the predicates of a language which assign the ‘correct’ truth-values to the sentences in all possible worlds, no matter how thesecorrecttruthvalues are singled out» («что всегда имеется бесконечно много различных интерпретаций языка, который усваивает предложениям “корректные” значения истинности во всех возможных мирах, совершенно безотносительно к тому, каким образом эти “корректные” значения истинности выбираются») (Putnam 1981, 34–35, — курсив Патнема). Помимо строгого формального доказательства, теорема Патнема сопровождалась примером, в котором предложение «Кошка на коврике» оказывалось — но по всем правилам формальной семантики Фреге–Монтегю — означающим вишню на дереве. Теорема показывает, что невозможно, закрепив значения истинности предложений и сделав это даже во всех возможных мирах, тем самым закрепить и референцию. Референция не удерживается фрегевской семантикой, хотя многие современные специалисты по формальной семантике даже и после теоремы Патнема не хотят задумываться об этой истине. Использованный Патнемом способ доказательства его теоремы может быть полезен для дальнейшего изучения как поэтических тропов, так и божественных имен.

Если денотатом слова «кошка» может оказаться вишня, то, возможно, это влечет катастрофические последствия для фрегевского понимания референции (именно так считал Дэвид Льюис, который отождествлял это понимание с «нашей теорией познания» (Lewis 1984, 221)), но может быть вполне подходящим для поэзии и даже для чуть более чем обычно экспрессивного рода обыденной речи. Например, кличка Вишенка очень популярна для кошек (во всяком случае, в России). Вероятно, часто ее дают по причине внешнего сходства свернувшегося клубком котенка и вишни, и тогда смысл этой клички метафорический. Но можно представить себе и метонимический механизм возникновения такой же клички: например, если вишни — любимая игрушка котенка. Предложения, содержащие поэтические тропы, имеют значения истинности, которые не являются специфически «поэтическими значениями истинности», а значит, у них те же самые значения истинности, о которых говорят Фреге и Патнем. Таким образом, «кошка» может означать «вишню» («Вишенку»), и это будет вполне корректная референция. Соответствующие предложения будут вполне понятными и осмысленными.

Эти примеры говорят нам больше, чем примеры Куайна (обсуждавшего референцию терма «кролик», который может относиться и к животному, и к части этого животного в виде еды на тарелке и т. д.), поскольку Куайн брал свои примеры из уже готового словаря — либо реального, либо хотя бы воображаемого (cp.: Quine 1960). Наши примеры с кошками, которые оказываются вишнями, порождались метафорой или метонимией, т. е. и не из словарей и не из прагматики (вопреки Дэвидсону), но из других возможных миров посредством необычных смысловых линий. В таких случаях предложения, подобные «кошка — это вишня», не только истинны, но и осмыслены, хотя и отнюдь не во фрегевском смысле. Впрочем, от кошки Патнема наша кошка отличается тем, что ее к нам привели смысловые линии Хинтикки и Санду, а не интенсионалы Монтегю (которые суть мировые линии), использованные Патнемом при доказательстве его теоремы[54].

Зато божественные имена оказываются ближе к кошке Патнема, чем к кошкам метафоры и метонимии. Они ведь тоже относятся к интенсионалам и к индивидуумам, а не к свойствам и смысловым линиям. Поэтому, в отличие от метафоры и метонимии, божественные имена причастны к онтологии. Тем не менее их структурно-логическое сходство с поэтическими тропами настолько велико, что их хочется назвать «онтологизированными» поэтическими тропами. Оно и неудивительно: и поэтические тропы, и божественные имена, взятые с земной, культурно-исторической точки зрения, имеют общего предка — магическое слово. Грубо говоря и не вдаваясь в детали, можно сказать, что поэзия произошла из магии…

Ниже в таблице 1 дается сравнение логических структур обыкновенных дескрипций, тропов (метафоры и метонимии) и магических слов / божественных имен; при этом онтологические и прагматические различия между магическими словами и божественными именами не учитываются.

Первый столбец таблицы 1 представляет собой континуум. Это очевидно из следующего. С одной стороны, некоторые очень слабые и особенно так называемые «стершиеся» (по-английски «dead») метафоры почти неотличимы от обыкновенных дескрипций. С другой стороны, возможные миры художественной литературы и поэзии имеют некоторую специфическую онтологию, которая не является абсолютно нереальной и имеет много общего с тем миром (или тут лучше сказать «мирами»?), в котором живут его авторы и читатели[55]. Поэтому нельзя провести слишком строго границу между дескрипциями, поэтическими тропами и магическими словами / божественными именами.

Наиболее важные различия состоят в следующем. Обыкновенная дескрипция устанавливает бинарные отношения внутри одного мира, тогда как поэтические тропы и магические слова / божественные имена устанавливают некоторые межмировые связи. Однако логическая природа этих связей разная в случаях поэтических тропов, с одной стороны, и магических слов / божественных имен, с другой. В первом случае связи основаны на идентичности некоторых свойств, а во втором — на идентичности самих индивидуумов. Именно наличие межмировых связей делает интенсиональность нередуцируемой.

Таблица 1

Логическая структура Нередуцируемая интенсиональность Межмировая связь Причастность к онтологии
Дескрипция Бинарные отношения    –   –
Троп Смысловая линия    + +   –
Магическое слово /
Божественное имя
Мировая линия    + +   +

10. Именуя Неименуемое: Принципы дополнительности и соответствия

Предложения (2) и (3) устанавливают отношения идентичности между Богом и всем вообще и одновременно между Богом и ничем, что может быть эксплицировано в записи (8) и (9) (предложение (9) следует из (8) согласно NFA):

(8)  божественность ≡〚ni〛∧ божественность ≡ ¬〚ni

(9)  божественность ↔〚ni〛∧ божественность ↔ ¬〚ni

где〚ni〛— денотат божественного имени ni.

Левые части обеих конъюнкций, взятые изолировано, означали бы пантеизм, а правые — тоже взятые изолировано — атеизм, но Дионисий заявляет и левые, и правые предложения только внутри параконсистентных конъюнкций. Подобное использование структур классической логики внутри параконсистентных структур мы уже назвали выше Дионисиевым принципом соответствия, аналогичным принципу соответствия в квантовой механике (где собственно квантовые концепции только «соответствуют» классическим, сами не будучи классическими).

Также бросается в глаза, что (8) и (9) воплощают принцип дополнительности (две взаимоисключающие — в классической логике — концептуальные схемы применяются одновременно параконсистентным способом), хотя для (8) и (9) имеются аутентичные названия — катафатическое богословие и апофатическое богословие. Оба этих богословия не существуют порознь, и одно никоим образом не «выше» другого. В терминах современной квантовой логики[56] можно сказать, что они «соответствуют» (в смысле принципа соответствия) классической логике предикатов.

Конечно, для понимания Дионисия совершенно необязательно занять какую-то определенную позицию в спорном вопросе интерпретации квантовой теории. Тем не менее те виды квантовой логики, которые формализуют подход Нильса Бора, т. е. Копенгагенскую интерпретацию квантовой теории, независимо от своей применимости в физике являются особенно полезными для нашей цели — понимания Ареопагита и византийской патристической традиции.

Вернемся к конъюнкциям (8) и (9). «Пантеистические» части конъюнкций указывают на божественный уровень онтологии тварного бытия, т. е. они указывают на тот факт, что все тварное бытие обожжено божественными логосами. Это обожение неотделимо от тварных индивидуумов, классов и даже ситуаций (поскольку в последних все равно действует божественное Провидение, т. е. те же логосы). «Атеистические» части конъюнкций указывают на абсолютное отличие между всем тварным бытием и бытием божественным — отличие настолько полное, что у них нет и не может быть общей онтологии.

Дионисий формулирует свои «неклассические» (т. е. не аристотелевские) идеи на «классическом» языке, т. е. на общем языке платонического аристотелизма своей эпохи. Поэтому его логика сформулирована как расширение классической — аналогично тому, как принцип соответствия был сформулирован Нильсом Бором для физики. Дионисиева логическая мысль «неклассическая», но его логический язык остается вполне обычным языком классической логики.

Что касается принципа дополнительности, то он не только эксплицируется в конъюнкциях (8) и (9), но и имплицирован внутри каждой из них. В качестве иллюстрации рассмотрим знаменитую фразу Дионисия: «Ὁ θεῖος γνόφος ἐστὶ τὸ “ἀπρόσιτον φῶς”, ἐν ᾧ κατοικεῖν ὁ θεὸς λέγεται» («Божественный мрак есть неприступный свет, в нем же речется Бог жити (1 Тим. 6:16)» (Epistolae 5.1; 162.3–4/1073A).

Формально «мрак» не является отрицанием к «свет», но это предложение (общий смысл которого состоит в том, что Бог — это сразу и свет, и мрак) содержит осмысленное противоречивое выражение, т. е., еще раз, — оно построено в соответствии с принципом дополнительности.

Говоря в общем, принцип дополнительности выражается формально следующим образом[57]. Пусть Т будет некая логическая теория, а α и β формулы ее языка, т. е. языка теории Т. Тогда можно сказать, что α и β суть Т-дополнительны (дополнительны в теории Т), если существует такая формула γ языка Т, что:

(10)  

Из этого определения следует, что α и ¬α также дополнительны друг к другу. Важным свойством (10) является следующая теорема: если α и β суть теоремы T, то γ ∧ ¬γ в общем случае не является теоремой T. Отсюда видно, что Т может быть классической теорией, которая не содержит внутренней параконсистентности. Например, «частица» не является отрицанием «волны», и как частицы света (фотоны), так и световые волны могут быть продемонстрированы экспериментально и описаны средствами классической физики (основанной на классической логике), но квантовая теория требует, чтобы оба таких описания вместе образовывали конъюнкцию в рамках одной общей картины, где эти описания приводят к противоречию (свет является частицами и не является частицами, он является волнами и не является волнами). Тут противоречие принадлежит квантовой теории, тогда как описания рассеяния фотонов и дифракции света, взятые по отдельности, принадлежат классической физике.

В нашем примере из Дионисия Бог («божественный мрак») является мраком, но Он — также и свет. Эти описания не обязательно взаимоисключающи: например, какой-нибудь другой объект — скажем, маяк — может быть попеременно мраком и светом, однако они взаимоисключающи и одновременно истинны в данном контексте у Дионисия. Каждое из двух описаний («мрак» и «свет») находится в пределах классической логики, но их неклассическая конъюнкция параконсистентна (дополнительна).

11. Именуя Неименуемое: возникновение интенсиональности

Мы достигли того момента, когда можно сделать некоторое промежуточные выводы. Они будут касаться того, откуда возникает параконсистентность божественных имен. Это объяснит нам, почему божественные имена нельзя интерпретировать ни как дескрипции, ни как жесткие десигнаторы, хотя они имеют нечто общее с теми и другими.

Божественные имена роднит с дескрипциями ровно то, что роднит их с поэтическими тропами. Тропы основаны на сходстве свойств (метафора) или отношениях смежности (метонимия). Также и божественные имена некоторым образом «описывают» Бога через некие свойства (которые, впрочем, не являются непосредственно свойствами божественными: последними являются только сами нетварные логосы). Но обычным дескрипциям не свойственна параконсистентность, а применительно к Богу она неизбежна; впрочем, поэтическим тропам она тоже свойственна. Во всяком случае, ясно, что интерпретация божественных имен как дескрипций совершенно недостаточна. Тогда можно, оставаясь все еще в пределах классической логики (хотя и многомировой семантики), задуматься о сходстве божественных имен с жесткими десигнаторами в смысле Крипке. Жесткие десигнаторы — это «ожестевшие» (употреблю это церковнославянское слово) дескрипции, и этим они отличаются от «token» («ярлыков») Баркан Маркус (эти «ярлыки» суть знаки, обладающие прямой десигнацией, т. е. отсылкой к денотату без коннотации, не будучи никак связаны с денотатом по своему смыслу, т. е. коннотату).

Теория жестких десигнаторов как «ожестевших» дескрипций была не так давно разработана Скоттом Соамесом (Soames 2002, 110–130, cp.: ibid., 49)[58], который ввел для них термин «частные дескриптивные имена» («partial descriptive names»). Соамес также сравнил эти имена с результатом применения к дескрипциям логического оператора Dthat Дэвида Каплана (Kaplan 1978) (Dthat превращает дескрипцию D в «dthat D», что является просто денотатом D, зафиксированным для любого контекста, в том числе даже для таких миров, в которых дескрипция D не имеет денотата).

Однако и частные дескриптивные имена Соамеса, и «dthat D» Каплана подразумевают одноуровневую онтологию, поэтому даже такой подход не будет вмещать божественных реалий, а именно одновременных описуемости и неописуемости Бога. Логика поэтических тропов в духе Хинтикки — Санду[59] тут подошла бы больше, поскольку ее многомировая семантика, в принципе, годится для представления многоуровневой онтологии[60]. Если добавить к теории поэтических тропов Хинтикки и Санду наше наблюдение о присущей тропам параконсистентности, то это объяснит причину нередуцируемости интенсиональности в поэтическом языке и, следовательно, отчасти и в божественных именах. Но для божественных имен есть и более специфическая, хотя тоже не богословская причина для нередуцируемости интенсиональности.

Логически сходная проблема решается в квантовой логике, выражающей идеи Копенгагенской интерпретации квантовой теории (далеко не случайно, что основные идеи этой логики были вдохновлены работами по формальной семантике естественных языков). Переход от формальной семантики языка к квантовой логике был совершен очень просто: через установление эквивалентности между пропозициями и множествами возможных миров, где эти пропозиции истинны, т. е. где исходы физических событий подтверждают данную пропозицию)[61]. Нередуцируемость интенсиональности в таких системах следует непосредственно из неразличимости квантовых объектов, где разные индивидуумы демонстрируют абсолютно одинаковые свойства, включая даже спатио-темпоральную локализацию (тем самым опровергая так называемый принцип Лейбница о тождестве неразличимых — от которого, впрочем, сам Лейбниц успел отказаться в последние месяцы своей жизни)[62].

Божественные логосы внутри тварных существ образуют предельный случай подобного соединения отчетливо индивидуализированных объектов, которые соответствуют каждый своему тварному существу, но также и единому Богу. Логосы различны и индивидуальны, но не имеют различия свойств, и одновременно они не только не различны, но даже едины, причем едины даже более, чем доллары на знаменитом банковском счете Шрёдингера[63].

Различие между логосами становится наблюдаемым только через их тварные ἔργα («дела, сделанное»), т. е. через разнообразие тварного мира[64]. Это напоминает ситуацию с двумя электронами, которые могут быть экспериментально посчитаны как два разных объекта, но которые не могут быть упорядочены, т. е. им не могут быть присвоены порядковые номера «первый», «второй», так как они неразличимы. Логическая природа множественности в Боге, выражающаяся божественными логосами, имеет сходную логическую природу: это кардинальность, никогда не переходящая в ординальность, поскольку логосы (нетварные энергии) неразличимы, или, другими словами, они различимы только по числу, но не по порядку.

Таковы основополагающие логические идеи, стоящие за Дионисиевым богословским учением о познании Бога как обожении — актах «экспериментального» претерпевания (παθῶν) божественного (DN 2.9, 134.2/648B).

12. Дескрипционизм Василия Великого  

Дионисий формулировал свое учение об именуемости Бога во второй половине V в., когда наиболее разработанным и наиболее известным христианским учением по этой теме было учение Каппадокийцев, особенно в том виде, в котором Василий Великий и Григорий Нисский сформулировали его в ходе полемики против Евномия[65].

Согласно распространенному среди ученых и, скорее всего, справедливому убеждению, именно это учение стало основой для Дионисиева учения о божественных именах[66]. Однако имелось существенное различие в целях: каппадокийское учение было направлено на обоснование внутритроичных различий. Поэтому Каппадокийцы лишь в общих словах касались того, чем Дионисий занят по преимуществу, — проблемы таких божественных имен, которые относятся к Богу, но не учитывают различие ипостасей. В своей трактовке нетриадологических имен Божиих Каппадокийцы имели весьма ограниченную цель — доказать, что имена, заимствованные из обыденного языка, могут быть отнесены к Богу хоть и не в буквальном смысле, но и не так, как если бы при отнесении к Богу на месте любого данного слова возникал его омоним (т. е. слово, имеющее тот же самый вид, но совершенно другой смысл). В такие подробности, которые будут важны для Дионисия, Каппадокийцы не входили. Основная часть их теории имени — представление об именах индивидуальных существ как отличных от имен общих сущностей (классов). В отличие от Дионисия, Каппадокийцы были обязаны построить логическую теорию собственных имен для каждой из ипостасей Троицы.

Исследования теории имени у Каппадокийцев находятся пока на начальной стадии. Подробно изучены только труды Василия Великого. Столь же подробное изучение трудов Григория Нисского — а также, по всей видимости, трудов таких тесно связанных с Каппадокийцами авторов, как Маркелл Анкирский и Дидим Слепец, — все еще остается в списке desiderata. Разумеется, речь идет не о простом пересказе того, что написано этими авторами, а об анализе, сделанном с учетом современных семантических теорий. Что касается исследований, посвященных семантике имен у Василия Великого, то они сосредоточены на проблемах собственных имен людей и на тех теориях античной Греции, на которые опирался в этом Василий. Однако собственно Василиева логика начинается там, где эти человеческие имена оказываются приложимы к ипостасям Святой Троицы, но на этом пороге исследователи логики и семантики Василия — и без того не особенно многочисленные — всегда останавливаются[67].

Тем не менее в некоторых недавних исследованиях, по крайней мере, признано наличие проблемы «напряжения» между использованием Василием Великим обычной для его эпохи философской терминологии и естественного языка, с одной стороны, и применением того и другого к Троице, с другой. Но даже в этих исследованиях Василиева процедура «очищения» (καθαρεύειν — его термин) соответствующих понятий прежде их применения к Богу лишь описана со слов самого Василия, но без какого-либо логического анализа[68].

Василий должен был доказать (против Евномия), что термин ἀγεννητός («нерожденный»), относимый к Отцу как арианами, так и их оппонентами, относится лишь к ипостаси («ипостась» означает индивидуальное бытие, то, что близко к понятию «первой сущности» у Аристотеля), а не ко «второй сущности» по Аристотелю (универсальное, или общее бытие, т. е. роды и виды). Часть Василиева доказательства построена на его собственной семантике собственных имен. По крайней мере, пока что для нее неизвестны никакие прецеденты. Ключевым пассажем является Contra Eunomium II.4 (Sesboüé 1983, 18–22), где Василий объясняет свое понимание собственных имен людей как «конъюнкцию разных особенностей» — «ἑτέρων ἰδιωμάτων συνδρομή». Термин συνδρομή буквально означает «совместный (за)бег», но в качестве философского термина он означает конъюнкцию. Такая конъюнкция образует специфический «характúр» (χαρακτήρ) каждого человека. (Термину «характúр» предстоит великое будущее в спорах о иконопочитании — где он превратится в важнейший христологический термин[69], — в связи с чем лучше его не переводить, а оставить как есть.) Поэтому собственные имена относятся не к человеческому роду в целом, а только к человеческим индивидуумам.

Как отметили и Каллигас, и Робертсон, подход Василия здесь очень похож на способ, найденный Джоном Сёрлем, чтобы обойти сразу и теорию жестких десигнаторов Крипке, и экстернализм Патнема (который отрицает всякую внутреннюю связь между словами и их значениями)[70], возвращаясь — но уже на другом уровне — к дескрипционизму Фреге и Рассела. Согласно Сёрлю[71], имена собственные все-таки указывают на индивидуальные характеристики своих объектов, но делают это весьма расплывчато. Релевантные характеристики могут быть собраны в различные множества, и тогда любое имя собственное будет соответствовать логической сумме (инклюзивной дизъюнкции) различных дескрипций. Это означает, что существует несколько разных способов объяснить, кто такой Аристотель (пример из Сёрля), или, скажем, кто такие Петр и Павел (пример из Василия). Эти способы различны, но не взаимоисключающи: например, Аристотель — это и «человек родом из Стагиры», и «учитель Александра Македонского». Вот почему их дизъюнкция инклюзивна.

Все неподходящие контексты, в которых подобные дескрипции могли бы быть поняты неправильно, отбрасываются из процесса коммуникации посредством интенции (фундаментальная категория у Сёрля) общающихся личностей. Таким образом, фрегевский дескрипционизм остается в силе, но только при условии его помещения в контекст, ограниченный интенциональностью.

У Василия Великого собственные имена людей функционируют весьма сходным образом. Василиев χαρακτήρ также является логической суммой (инклюзивной дизъюнкцией) различных характеристик (индивидуальных идиом), т. е. множеством доступных дескрипций (или, как справедливо заметил Каллигас, счетным множеством таких дескрипций за вычетом пустого множества). Но самым главным остается вопрос о том, каким образом такую теорию можно применить к ипостасям Святой Троицы.

13. Именуя Неименуемое: метонимический подход

Итак, Василий Великий интерпретирует собственные имена ипостасей Троицы как дескрипции. Поэтому, например, имя «Отец» относится к «отцовству», т. е. к тому, чтобы быть нерожденным (ἀγεννητός) и чтобы рождать самому, а имя «Сын» относится к «сыновству», т. е. к тому, чтобы быть рожденным Отцом[72]. Но далее начинается различие между применением подобных имен к Троице и к людям. Для ипостасей Троицы никакого различия дескрипций не предусматривается, хотя одни и те же дескрипции могут выражаться в разных словах (например, можно сказать «отцовство», а можно сказать «быть нерожденным и рождать», но это одно и то же по экстенсиональному содержанию). Если вспомнить сёрлевский пример с Аристотелем, то там разные дескрипции экстенсионально различны: «человек родом из Стагиры» экстенсионально отличается от «учителя Александра Македонского».

Для каппадокийского богословия совершенно принципиально, что идиома, отличающая одну (любую) из ипостасей Троицы, единственная. Она может быть описана разными словами, но она экстенсионально тождественна самой себе. Получается, что соответствующее счетное множество дескрипций для каждой из ипостасей Троицы превращается в синглетон — множество из одного элемента. Для трех ипостасей существуют только три идиомы, по одной на каждую.

Эти три идиомы имеют одну важную особенность: они являются свойствами отношений (τὸ πρὸς τί, что можно перевести как «отношение»), которые определяются друг через друга. Например, свойство отношения «быть отцом» подразумевает наличие другого существа, обладающего свойством отношения «быть сыном». Экстенсиональное содержание соответствующих термов, отнесенных к божественной реальности, остается таким же, как и в их обыденном употреблении применительно к тварным созданиям. Как формулирует это сам Василий, «…ибо различие между Сыном и всем остальным состоит не в отношении (понятие «отношение» само по себе тут не специфично. — В. Л.); напротив, превосходство Бога над смертными проявляется в том, что специфично для Его сущности»[73]. Здесь Василий эксплицитно формулирует принцип метонимии (он же принцип смежности, или отношения) как основу его концепции тринитарных божественных имен. Если сравнивать божественное отцовство с человеческим, то различие будет состоять не в самом этом принципе отношения, а лишь в самих объектах, связанных по этому принципу.

Таким образом, именование триединого Бога «Отцом», «Сыном» и «Духом» является некоторого рода метонимией, — но, конечно, в отличие от обыкновенной метонимии, тут подразумевается некое онтологическое содержание и мировые линии вместо смысловых линий.

Сами по себе имена «Отец», «Сын» и «Дух» могут быть обращены к Богу и метафорически, когда они не рассматриваются относительно друг друга. Таких примеров очень много, особенно в библейской традиции (правда, за естественным исключением имени «Сын»)[74]. Однако в подобных случаях эти имена не имеют того же самого смысла, что и в триадологическом контексте. В богословских дискуссиях по триадологическим вопросам (например, по поводу Filioque) в тех их частях, которые основаны на обсуждении Писания, мы очень часто встречаемся с этой необходимостью различить между «метафороподобным» и «метонимоподобным» употреблением соответствующих божественных имен, т. е. между случаями, когда они относятся к Богу вообще или когда они относятся к определенной ипостаси (классический пример — «дух» из «дуновения» Христа в Ин. 20:22 в сравнении с подаянием Духа в Пятидесятницу).

Таким образом, фундаментальное отличие внутритроичных имен Божиих от тех имен, которые относятся к Богу без различия ипостасей, состоит в том, что они определены как система трех относительных свойств, связанных друг с другом. Другими словами, триединый Бог описывается тремя дескрипциями, которые, в свою очередь, являются дизъюнкциями (логическими суммами) уникальных дескрипций (т. е. суммы состоят в каждом случае только из одного слагаемого).

Такой основанный на метонимии богословский язык не является совсем уж чуждым и для Дионисия, но Дионисий нигде не останавливается на нем подробно. Он лишь ограничивается одним и, по сути, единственным упоминанием о том, что этот язык ему известен, и он умеет им пользоваться:

Πάλιν, ὅτι μέν ἐστι πηγαία θεότης ὁ πατήρ, ὁ δὲ υἱὸς καὶ τὸ πνεῦμα τῆς θεογόνου θεότητος, εἰ οὕτω χρὴ φάναι, βλαστοὶ θεόφυτοι καὶ οἷον ἄνθη καὶ ὑπερούσια φῶτα, πρὸς τῶν ἱερῶν λογίων παρειλήφαμεν (DN 2.7, 132.1–3/645B)[75].

14. Тринитарный коннектив: параконсистентное тернарное эксклюзивное или

Чтобы завершить наше рассмотрение семантики каппадокийской идеи триединства, опишем более формальным образом ее логический скелет.

Пусть di обозначает одну из дескрипций, имплицируемых собственным именем p, так что само p может быть определено (с учетом правильного интенционального контекста) как инклюзивная дизъюнкция следующего вида:

(11)  

Однако, применяя такое определение к божественным ипостасям, мы вынуждены оставить только единственную дескрипцию, d. Таким образом, для собственных внутритроичных имен, P, F и S, дескрипции являются синглетонами dP, dF, и dS. Все они определены относительно друг друга, но при этом являются взаимоисключающими: обладать свойством dP означает не обладать ни свойством dF, ни свойством dS, и т. д.

Таким образом, для каждой из трех ипостасей только одно из этих трех свойств может быть и на самом деле является истинным. Другими словами, для каждой данной ипостаси эти три свойства соединяются тернарным эксклюзивным коннективом или, который не следует путать с эксклюзивной дизъюнкцией в обычном смысле этого слова, ⊕. Тернарное эксклюзивное или (которое мы будем обозначать, вслед за Пеллетье и Хартлайном (Pelletier, Hartline 2008), ⊻3) означает «в точности только один из трех»:

(12)  ⊻3 (dP, dF, dS) (для каждой из трех ипостасей).

Коннектив ⊻n может быть назван «the “real” variable adicity exclusive or, “because” …it is the one that is relevant to formal accounts of natural language» («“реально” эксклюзивным или переменной арности, поскольку …именно этот коннектив является релевантным для формальной репрезентации естественного языка»), хотя, увы, «…that topic has not been addressed by the logic textbooks (nor by the formal semantic descriptions of natural language)» («…эта тема не рассматривается в учебниках логики (и не рассматривается даже в работах по формальной семантике естественных языков)») (ibid., 77)[76]. Вместо этого учебники логики уделяют много внимания другому виду эксклюзивной дизъюнкции, значение которого становится совершенно иным при арности больше двух.

Эта обыкновенная эксклюзивная дизъюнкция ⊕ (грубо соответствующая «или… или» естественных языков) является бинарным, а не тернарным коннективом. Поэтому ⊕3 (d1, d2, d3) = d1d2d3, что является просто итерацией ⊕. Различие между двумя коннективами, ⊻ и ⊕, критично для понимания тринитарной логики Каппадокийцев.

В едином Боге три ипостасных свойства образуют тернарный кондиционал:

(13)  dPdFdS (для единого Бога),

что означает, что существование (в каком бы то ни было смысле слова) любой из ипостасей подразумевает существование (в том же смысле слова) двух остальных; отдельное существование ни для одной из ипостасей невозможно.

В классической логике не составляло бы проблемы атрибутировать одному отдельному логическому объекту (12), а другому отдельному логическому объекту (13). Но у нас, увы, нет такой пары разных логических объектов. Вместо нее мы имеем только один и единственный логический объект, который является одновременно и единым Богом, и тремя ипостасями. Мы получаем знаменитое параконсистентное равенство каппадокийской логики: 3 = 1 (утверждение о том, что Бог триедин). К этому же параконсистентному равенству можно подойти, исходя из анализа только онтологии, а точнее, категорий первой и второй сущностей в контексте Ареопагита (Лурье 2006, 80–101), но мы сейчас подошли к нему со стороны семантики, начав с уяснения смысла дескрипций. Наши условия (12) и (13), если их наложить одновременно, образуют параконсистентный бикондиционал (14):

(14)  ⊻3 (dP, dF, dS) ↔ (dPdFdS)

Это выражение обозначает, что оба и только оба высказывания истинны одновременно: Бог — это Отец, Сын и Дух, но для Него невозможно быть Отцом, будучи Сыном или Духом.

Если бы вместо тернарного эксклюзивного или у нас был бы другой коннектив, ⊕3, то бикондиционал (14) утратил бы свою параконсистентность и оказался бы тривиальным. Бинарная эксклюзивная дизъюнкция арности n, ⊕n, где n — натуральное число, эквивалентна n-арному кондиционалу для любого нечетного n, начиная с n = 3. Это очевидно из того факта, что логический бикондиционал является отрицанием эксклюзивной дизъюнкции ⊕, и напротив: ab = ¬(ab). Поэтому для любого нечетного n ≥ 3 значение истинности функции ⊕n будет «истинно» не только в том случае, если только один из ее n аргументов истинен, но также и в случае, если все они истинны. Напротив, значение истинности функции ⊻n будет «истинно» только в том случае, если только один — а не все — из ее n аргументов будут истинны. Если принять это во внимание, то функцию ⊕n можно назвать «нечетной считающей функцией арности n», а итерации ⊕ следовало бы назвать «сложением с остатком от деления на 2», а не «эксклюзивной дизъюнкцией» (Pelletier, Hartline 2008, 77). ⊻n также является считающей функцией при n = 2, так как она принимает те же значения истинности, что и ⊕2 (т. е. обычная ⊕). Однако ⊻2 является считающей функцией только «экстенсионально» — а «интенсионально» она является производной от ⊻3 (причем обратное — неверно), а последняя не является считающей функцией: «iterating ⊻3 does not “keep track of” how many arguments are true / false» («итерации ⊻3 не “ведут записи” того, сколько ее аргументов истинны или ложны») (ibid., 80).

Поэтому применение для триадологии итерированной бинарной эксклюзивной дизъюнкции вместо тернарной дизъюнкции подразумевало бы «ведение записи» значений истинности ее аргументов, и, таким образом, выражало бы некий вид субординации между божественными ипостасями, — что является абсолютно недопустимым в рамках богословия Каппадокийцев.

Кроме того, при n = 3, ⊕3 (d1, d2, d3) = d1 d2d3, что сделало бы наш бикондиционал (14) тривиальным, если бы вместо ⊻3 он содержал ⊕3.

15. Заключение

Мы постарались избежать — настолько, насколько это было возможно, т. е. не вполне, — обсуждения онтологии и эпистемологии Ареопагита. Вместо этого мы сосредоточились на семантике. В каком смысле Дионисиевы «божественные имена» являются настоящими именами, а в каком смысле это просто дескрипции? В каком смысле неименуемый Бог становится именуемым через эти божественные имена?

Как следовало ожидать, семантика Ареопагита оказалась параконсистентной, но зато в некоторых отношениях она оказалась ближе к классической логике, чем можно было предполагать. Дионисиевы «божественные имена» можно рассматривать как предельный случай поэтического тропа, но при этом включающего онтологическое содержание. Соответствующая семантика нередуцируемо интенсиональна и не-фрегева. В современной логике ближайшие параллели к логике и семантике Ареопагита можно найти в тех вариантах квантовой логики, где используется многомировая семантика Крипке и нарушается принцип идентичности неразличимых Лейбница.

Дионисиева неклассическая логика тесно связана с классической логикой через принцип соответствия, а параконсистентность вводится у Дионисия через принцип дополнительности. Это, разумеется, не инновация самого Дионисия, а следование уже сложившейся к его времени традиции — главным образом идущей от Каппадокийцев. Для обоих только что названных принципов я сознательно использовал те названия, которые им дал Нильс Бор, так как именно он ввел их в современные физику и философию.

За пределами Ареопагита и богословия вообще нередуцируемо интенсиональная семантика поэтических тропов представляет самостоятельный интерес. Метафора и метонимия не могут быть полностью переведены на язык дескрипций: любой пересказ их обычными словами разрушил бы именно тот компонент значения, ради которого и был использован соответствующий поэтический троп. Тем не менее тропы нормативно используются для повышения объяснительной мощи дискурса, а не для напускания тумана. Это доказывает, что наше мышление работает не-фрегевскими способами. Поэтому подлинные законы правильного мышления, которые называются логикой, нередуцируемо интенсиональны, и их можно подчинить карнаповскому принципу экстенсиональности только в отдельных случаях. Такова была и базовая интуиция Лейбница, отца современных интенсиональных логик. Неудивительно, что та же самая интуиция обнаруживается в основе святоотеческой логики.

P. S. 2018 года

Нередуцируемость интенсиональности в случае святоотеческой логики связана, как я постарался показать в данной статье, с неконсистентностью, вполне эксплицитной, основных относимых к Богу понятий. Неконсистентностью, предполагающей все виды противоречий — субконтрарные, контрадикторные и контрарные, хотя контрарные мною в этом тексте почти не рассматривались. В мою задачу входило сделать акценты на постановке вопросов и определении тех направлений, в которых следует искать решения, каждое из которых должно стать предметом отдельного исследования. Отчасти такие исследования можно найти в моих более поздних работах (см.: Лурье 2016; Lourié 2018; Lourié 2019 (в печати)).

Список литературы

Barcan Marcus R. (1993) «Modalities and Intensional Languages» (1961–1967). Barcan Marcus R. Modalities. Philosophical Essays. New York; Oxford: Oxford University Press. P. 3–38.

Barwise J. (1989) The Situation in Logic. Stanford, CA: Center for the Study of Language and Information, Stanford University (CSLI Lecture Notes. № 17).

Barwise J., Perry J. (1981) «Semantic Innocence and Uncompromising Situations». Midwest Studies of Philosophy. Vol. 6. № 1. P. 387–403 (Переиздание: The Philosophy of Language. Ed. by A. P. Martinich. New York; Oxford: Oxford University Press, 2001. P. 392–404).

Bealer G. (2000) «Intensional entities». Routledge Encyclopedia of Philosophy. Vol. 4. Ed. by E. Craig. London: Routledge. P. 803–806.

Bohr N. (1958) Atomic Physics and Human Knowledge. New York: John Wiley & Sons.

Carnap R. (1947) Meaning and Necessity. A Study in Semantics and Modal Logic. Chicago: University of Chicago Press.

Carnap R. (2001) The Logical Syntax of Language (1937). Transl. by A. Smeaton (Countess von Zeppelin). London: Routledge (International Library of Psychology).

Cattaneo G., Dalla Chiara M. L., Giuntini R., Paoli F. (2009) «Quantum Logic and Nonclassical Logics». Handbook of Quantum Logic and Quantum Structures: Quantum Logic. Ed. by K. Engesser, D. M. Gabbay, D. Lehmann. Amsterdam: Elsevier. P. 127–226.

Catterson T. (2004) «Hintikka on the Problem with the Problem of Transworld Identity». Quantifiers, Questions and Quantum Physics. Essays on the Philosophy of Jaakko Hintikka. Ed. by D. Kolak, J. Symons. Dordrecht: Springer. P. 33–47.

Chalmers D. J. (2002) «On Sense and Intension». Philosophical Perspectives 16: Language and Mind. Ed. by J. E. Tomberlin. Oxford: Blackwell. P. 135–182.

Chalmers D. J. (2010) The Character of Consciousness. Oxford: Oxford University Press (Philosophy of Mind).

Chernoff F. (1981) «Leibniz’s Principle of Identity of Indiscernibles». The Philosophical Quarterly. Vol. 31. № 123. P. 126–138.

Church A. (1951) «A Formulation of the Logic of Sense and Denotation». Structure, Method, and Meaning: Essays in Honor of Henry M. Scheffer. Ed. by P. Henle, H. M. Kallen, S. K. Langer. New York: The Liberal Arts Press. P. 3–24.

Corsini E. (1962) Il trattato «De Divinis nominibus» dello Pseudo-Dionigi e i commenti Neo-Platonici al «Parmenide». Torino: G. Giappichelli (Università di Torino. Pubblicazioni della Facoltà di lettere e filosofia. Vol. 13. Fasc. 4).

Dalla Chiara M. L. (1987) «An Approach to Intensional Semantics». Synthese. An International Journal for Epistemology, Methodology and Philosophy of Science. Vol. 73. № 3. P. 479–496.

Dalla Chiara M. L., Toraldo di Francia G. (1992) «Individuals, Kinds, and Names in Physics». Bridging the Gap: Philosophy, Mathematics, and Physics. Ed. by G. Corsi, M. L. Dalla Chiara, G. C. Ghirardi. Dordrecht: Kluwer Academic Publishers (Boston Studies in the Philosophy of Science. Vol. 140). P. 261–283.

Dalla Chiara M. L., Giuntini R., Krause D. (1998) «Quasi set Theories for Microobjects: a Comparison». Interpreting Bodies: Classical and Quantum Objects in Modern Physics. Ed. by E. Castellani. Princeton: Princeton University Press. P. 142–152.

Dalla Chiara M. L., Giuntini R., Greechie R. (2004) Reasoning in Quantum Theory. Sharp and Unsharp Quantum Logics. Dordrecht: Kluwer Academic Publishers (Trends in Logic. Studia Logica Library. Vol. 22).

Davidson D. (1991a) «True to the Facts» (1969). Davidson D. Philosophical Essays. Vol. 2. Inquiries into Truth and Interpretation. Oxford: Clarendon Press. P. 37–54. (Первое издание: Oxford: Clarendon Press, 1984).

Davidson D. (1991b) «What Metaphors Mean» (1978). Davidson D. Philosophical Essays. Vol. 2. Inquiries into Truth and Interpretation. Oxford: Clarendon Press. P. 245–264

Davidson D. (2001) «The Myth of the Subjective» (1988). Davidson D. Philosophical Essays. Vol. 3. Subjective, Intersubjective, Objective. Oxford: Clarendon Press. P. 39–52.

Del Cogliano M. (2010) Basil of Caesarea’s Anti-Eunomian Theory of Names. Christian Theology and Late-Antique Philosophy in the Fourth Century Trinitarian Controversy. Leiden; Boston: Brill (Supplements to Vigiliae Christianae. Vol. 103).

Doležel L. (1998) Heterocosmica. Fiction and Possible Worlds. Baltimore; London: The Johns Hopkins University Press (Parallax: Re-visions of culture and society).

Drecoll V. H. (1996) Die Entwicklung der Trinitätslehre des Basilius von Cäsarea. Sein Weg vom Homöusianer zum Neonizäner. Göttingen: Vandenhoeck & Ruprecht (Forschungen zur Kirchen- und Dogmengeschichte. Bd. 66).

Edel S. (1996) «Métaphysique des idées et mystique des lettres: Leibniz, Böhme et la Kabbale prophéthique». Revue de l’histoire des religions. T. 213. № 4. P. 443–466.

Engström A. (2001) «Hintikka and Sandu on Metaphor». Philosophia. Philosophical Quarterly of Israel. Vol. 28. № 1–4. P. 391–410.

Frege G. (1892) «Über Sinn und Bedeutung». Zeitschrift für Philosophie und philosophische Kritik. Bd. 100. S. 25–50.

French S., Krause D. (2006) Identity in Physics: A Historical, Philosophical, and Formal Analysis. Oxford: Oxford University Press.

Goldblatt R. I. (1974) «Semantic Analysis of Orthologic». Journal of Philosophical Logic. Vol. 3. № 1–2. P. 19–35.

Golińska-Pilarek I., Huuskonen T. (2005) «Number of Extensions of Non-Fregean Logics». Journal of Philosophical Logic. Vol. 34. № 2. P. 193–206.

Golitzin A. (1994) Et introibo ad altare Dei. The Mystagogy of Dionysius Areopagita, with Special Reference to Its Predecessors in the Eastern Christian Tradition. Θεσσαλονίκη: Πατριαρχικὸν Ἵδρυμα Πατερικῶν Μελετῶν (Ἀνάλεκτα Βλατάδων. Т. 59).

Golitzin A. (2007) «Dionysius Areopagites: A Christian Mysticism?». Scrinium. Journal of PatrologyCritical Hagiographyand Ecclesiastical History. Vol. 3. P. 128–179.

Heil G., Ritter A. M., hg. (1991) Corpus Dionysiacum II: Pseudo-Dionysius Areopagita. De coelesti hierarchia. De ecclestiastica hierarchia. De mystica theologia. Epistulae. Berlin; New York: Walter de Gruyter (Patristische Texte und Studien. Bd. 36).

Hildebrand S. M. (2007) The Trinitarian Theology of Basil of Caesarea: A Synthesis of Greek Thought and Biblical Truth. Washington, DC: Catholic University of America Press.

Hintikka J. (1974) «Knowledge and the Known». Historical Perspectives in Epistemology. Dordrecht; Boston: D. Reidel. P. 202–207 (Synthese Historical Library. Vol. 11).

Hintikka J. (1989) «On the Proper Treatment of Quantifiers in Montague Semantics» (1974). Hintikka J., Hintikka M. B. P. The Logic of Epistemology and the Epistemology of Logic: Selected Essays. Dordrecht: Kluwer Academic Publishers (Synthese Library. Vol. 200). P. 97–112.

Hintikka J. (1998) Selected Papers. Vol. 4: Paradigms for Language Theory and Other Essays. Dordrecht: Kluwer Academic Publishers.

Hintikka J., Sandu G. (1994) «Metaphor and Other Kinds of Nonliteral Meaning». Aspects of Metaphor. Ed. by J. Hintikka. Dordrecht: Kluwer Academic Publishers (Synthese Library. Vol. 238). P. 151–188.

Hintikka J., Sandu G. (1995) «The Fallacies of the “New Theory of Reference”». Synthese. An International Journal for Epistemology, Methodology and Philosophy of Science. Vol. 104. № 2. P. 245–283.

Jakobson R. (1971) «Two Aspects of Language and Two Types of Aphasic Disturbances» (1956). Jakobson R. Selected Writings. Vol. II: Words and Language. The Hague; Paris: Mouton. P. 239–259.

Kalligas P. (2002) «Basil of Caesarea on the Semantics of Proper Names». Byzantine Philosophy and Its Ancient Sources. Ed. by K. Ierodiakonou. Oxford: Oxford University Press. P. 31–48.

Kaplan D. (1978) «DTHAT». From Syntax to Semantics. Vol. 9: Pragmatics. Ed. by P. Cole. New York: Academic Press. P. 221–253 (Переиздание: The Philosophy of Language. Ed. by A. P. Martinich. New York; Oxford: Oxford University Press, 2001. P. 325–338).

Kaplan D. (1979) «Transworld Heir Lines». The Possible and the Actual. Ed. by M. Loux. Ithaca: Cornell University Press. P. 88–109.

Kittay E. F. (1995) «Metaphor as Rearranging the Furniture of the Mind: A Reply to Donald Davidson’s “What Metaphors Mean”». From a Metaphorical Point of View. A Multidisciplinary Approach to the Cognitive Content of Metaphor. Ed. by Z. Radman. Berlin; New York: Walter de Gruyter (Philosophie und Wissenschaft. Bd. 7). P. 73–116.

Knepper T. D. (2008) «Not Not: The Method and Logic in Dionysian Negation». American Catholic Philosophical Quarterly. Vol. 82. № 4. P. 619–637.

Knepper T. D. (2013) «Techniques and Rules of Ineffability in the Dionysian Corpus». Logic in Orthodox Christian Thinking. Ed. by A. Schumann. Heusenstamm bei Frankfurt: Ontos-Verlag. P. 122–172.

Krause D., da Costa N. C. A. (2004) «Complementarity and Paraconsistency». Logic, Epistemology, and the Unity of Science. Vol. 1. Ed. by H. Rahman, J. Symons, D. M. Gabbay, J.-P. Van Bendegem. Berlin; New York: Springer. P. 557–568.

Krause D., da Costa N. C. A. (2006) «The Logic of Complementarity». The Age of Alternative Logics: Assessing Philosophy of Logic and Mathematics Today. Ed. by J.-P. Benthem, G. Heinzmann, M. Rebuschi, H. Visser. Berlin; New York: Springer. P. 103–120.

Kripke S. (1963) «Semantical Considerations on Modal Logic». Acta Philosophica Fennica. Fasc. XVI. P. 83–94.

Kripke S. (1980) Naming and Necessity. Cambridge, MA: Harvard University Press (Переиздание: Cambridge, MA: Harvard University Press, 2001).

Krivochéine B. (1985) «Simplicité de la nature divine et les distinctions en Dieu selon S. Grégoire de Nysse». Studia Patristica. Vol. 16. P. 389–411.

Larchet J.-C. (2010) La théologie des énergies divines. Dès origines à saint Jean Damascène. Paris: Éditions du Cerf (Cogitatio Fidei. № 272).

Leezenberg M. (2001) Contexts of Metaphor. Amsterdam: Elsevier (Current Research in the Semantics. Pragmatics Interface. Vol. 7).

Lepore E., Stone M. (2010) «Against Metaphorical Meaning». Topoi. An International Review of Philosophy. Vol. 29. № 2. P. 165–180.

Lewis D. K. (1984) «Putnam’s Paradox». Australasian Journal of Philosophy. Vol. 62. № 3. P. 221–236.

Lewis D. K. (1986) On the Plurality of Worlds. Oxford: Blackwell.

Lossky V. (1930) «La notion des “analogies” chez Denys le pseudo-Areopagite». Archives d’histoire doctrinale et littéraire du Moyen Age. Vol. 5. P. 279–309.

Lourié B. (2000) «Le second iconoclasme en recherche de la vraie doctrine». Studia Patristica. Vol. 34. P. 145–169.

Lourié B. (2006) «Une dispute sans justes. Léon de Chalcédoine, Eustrate de Nicée et la troisième querelle sur les images sacrées». Studia Patristica. Vol. 42. P. 321–339.

Lourié B. (2010) «Peter the Iberian and Dionysius the Areopagite: Honigmann – van Esbroeck’s Thesis Revisited». Scrinium. Journal of PatrologyCritical Hagiographyand Ecclesiastical History. Vol. 6. Р. 143–212.

Lourié B. (2012) «Intensio: Leibniz in Creating a New Term for the Modal Logic». Studia Humana. Vol. 1. № 3–4. P. 59–65.

Lourié B. (2013) «Philosophy of Dionysius the Areopagite. Modal Ontology». Logic in Orthodox Christian Thinking. Ed. by A. Schumann. Heusenstamm bei Frankfurt: Ontos-Verlag. P. 230–257.

Lourié B. (2018) «A Logical Scheme and Paraconsistent Topological Separation in Byzantium: Inter-Trinitarian Relations according to Hieromonk Hierotheos and Joseph Bryennios». Relations. Ontology and Philosophy of Religion. Ed. by D. Bertini, D. Migliorini. [Sesto San Giovanni (Milano)]: Mimesis International (Mimesis International. Philosophy. № 24). P. 283–299.

Lourié B. (2019) «What Means ‘Tri-’ in ‘Trinity’? An Eastern Patristic Approach to the ‘Quasi-Ordinals’». Journal of Applied Logic. Vol. 6. № 6. P. 1093–1108 (в печати.)

Lourié B., Mitrenina O. (2015) «Semantic of poetical tropes: Non-Fregeanity and paraconsistent logic». Donum Semanticum. Opera linguistica et logica in honorem Barbarae Partee a discipulis amicisque Rossicis oblate. Ed. by P. Arkadiev, I. Kapitonov, Y. Lander, E. Rakhilina, S. Tatevosov, with assistance of P. Rudnev. M.: Languages of Slavic Culture. P. 177–191.

Lourié B., Mitrenina O. (2019) «The Role of Truth-Values in Indirect Meanings». Language, Music and Computing. Second International Workshop, LMAC 2017. Revised Selected Papers. Ed. by P. Eismont, O. Mitrenina, A. Pereltsvaig. Cham: Springer (Communications in Computer and Information Science. Vol. 943). P. 185–206. (в печати.)

Mao Y., Zhou B. (2007) «Interpreting Metaphors in a New Semantic Theory of Concept». New Frontiers in Artificial Intelligence. JSAI 2006 Conference and Workshops, Tokyo, Japan, June 5-9 2006, Revised Selected Papers. Ed. by T. Washio, K. Satoh, H. Takeda, A. Inokuchi. Berlin; Heidelberg: Springer (Lecture Notes in Computer Science. Lecture Notes in Artificial Intelligence. Vol. 4384). P. 177–190.

Mill J. S. (2001) «On Names». The Philosophy of Language. Ed. by A. P. Martinich. New York; Oxford: Oxford University Press. P. 266–271.

Montague R. (1974) Formal Philosophy. Selected Papers. Ed. by R. H. Thomason. New Haven; London: Yale University Press.

Mortley R. (1986) From Word to Silence. Vol. II. The Way of Negation, Christian and Greek. Bonn: Hanstein (Theophaneia. Vol. 31).

Quine W. V. O. (1960) Word and Object. Cambridge, MA: MIT Press.

Quine W. V. O. (1969) «Ontological Relativity» (1968). Quine W. V. O. Ontological Relativity and Other Essays. New York: Columbia University Press. P. 26–68.

Parsons T. (1967) «Grades of Essentialism in Quantified Modal Logics». Noûs. Vol. 1. № 2. P. 181–191.

Pelletier F. J., Hartline A. (2008) «Ternary Exclusive Or». Logic Journal of the IGPL. Vol. 16. № 1. P. 75–83.

Peltomaa L. M. (2001) The Image of the Virgin Mary in the Akathistos Hymn. Leiden; Boston; Koln: Brill (The Medieval Mediterranean. Vol. 35).

Perl E. D. (2007) Theophany. The Neo-Platonic Philosophy of Dionysius the Areopagite. New York: SUNY Press (SUNY series in ancient Greek philosophy).

Pietarinen A.-V. (2008) «An Iconic Logic of Metaphors». Proceedings of the 6th International Conference of Cognitive Science. Yonsei University: The Korean Society for Cognitive Science. P. 317–320.

Priest G. (1995) Beyond the Limits of Thought. Cambridge: Cambridge University Press.

Putnam H. (1979) «The meaning of ‘meaning’». Putnam H. Philosophical Papers. Vol. 2: Mind, Language and Reality. Cambridge: Cambridge University Press. P. 215–271.

Putnam H. (1981) «A Problem about Reference». Putnam H. Reason, Truth, and History. Cambridge: Cambridge University Press. P. 22–48, 217–218.

Putnam H. (2005) «A Philosopher Looks at Quantum Mechanics (Again)». The British Journal for the Philosophy of Science. Vol. 56. № 4. P. 615–634.

Radde-Gallwitz A. (2009) Basil of Caesarea, Gregory of Nyssa, and Transformation of Divine Simplicity. Oxford: Oxford University Press (Oxford Early Christian Studies).

Rhodes M. C. (2013) «On Contradiction in Orthodox Philosophy». Logic in Orthodox Christian Thinking. Ed. by A. Schumann. Heusenstamm bei Frankfurt: Ontos-Verlag. P. 82–103.

Robertson D. G. (2001) «Relatives in Basil of Caesarea». Studia Patristica. Vol. 37. P. 277–287.

Robertson D. G. (2002) «A Patristic Theory of Proper Names». Archiv für Geschichte der Philosophie. Bd. 84. Heft 1. P. 1–19.

Roques R. (1983) L’Univers dionysien. Structure hiérarchique du monde selon le Pseudo-Denys. Paris: Éditions du Cerf (Patrimoines Christianisme) (Первое издание: Paris: Aubier (Théologie), 1954).

Scazzoso P. (1967) Ricerche sulla struttura del linguaggio dello Pseudo-Dionigi Areopagita. Introduzione alla lettura delle opere pseudo-dionisiane. Milano: Vita e pensiero (Pubblicazioni dell’Università Cattolica «Sacro Cuore». Ser. III. Scienze sociali. Vol. 14).

Schäfer C. (2006) Philosophy of Dionysius the Areopagite. An Introduction to the Structure and the Content of the Treatise «On the Divine Names». Leiden; Boston: Brill (Philosophia Antiqua. Vol. 99).

Searle J. R. (1958) «Proper Names». Mind (New Series). Vol. 67. № 266. P. 166–173.

Searle J. R. (1983) Intentionality. An Essay in the Philosophy of Mind. Cambridge: Cambridge University Press.

Semino E. (1997) Language and World Creation in Poems and Other Texts. New York: Longman (Textual explorations).

Sesboiié B., Durand G.-M., Doutreleau L., éds. (1982) Basile de Césarée. Contre EunomeSuivi de EunomeApologie. Introduction, traduction et notes de B. Sesboiié, S.J., avec la collaboration pour le texte et l’introduction critiques de G.-M. de Durand, O.P., et L. Doutreleau, S.J. T. 1. Paris: Éditions du Cerf (Sources Chrétiennes. Vol. 299).

Sesboiié B., Durand G.-M., Doutreleau L., éds. (1983) Basile de Césarée. Contre EunomeSuivi de EunomeApologie. Introduction, traduction et notes de B. Sesboiié, S.J., avec la collaboration pour le texte et l’introduction critiques de G.-M. de Durand, O.P., et L. Doutreleau, S.J. T. 2. Paris: Éditions du Cerf (Sources Chrétiennes. Vol. 305).

Shani I. (2005) «Intension and Representation: Quine’s Indeterminacy Thesis Revisited». Philosophical Psychology. Vol. 18. № 4. P. 415–440.

Shani I. (2007) «The Myth of Reductive Extensionalism». Axiomathes. Vol. 17. № 2. P. 155–183.

Soames S. (2002) Beyond Rigidity: The Unfinished Semantic Agenda of Naming and Necessity. Oxford: Oxford University Press.

Stead G. C. (1988) «Logic and the Application of Names to God». El «Contra Eunomium I» en la Produccion Literaria De Gregorio De NisaVI Coloquio Internacional sobre Gregorio de Nisa. Edición a cargo de L. F. Mateo-Seco, J. L. Bastero. Pamplona: Ediciones Universidad de Navarra (Colección teológica). P. 303–320.

Stead C. (2000) Doctrine and Philosophy in Early Christianity. Arius, Athanasius, Augustine. Aldershot; Burlington; Singapore; Sydney: Ashgate (Variorum Collected Studies Series. Vol. 224).

Steel C. (2003) «Beyond the Principle of Contradiction? Proclus’ “Parmenides” and the Origin of the Negative Theology». Die Logik des Transzendentalen. Festschrift für Jan A. Aertsen zum 65. Geburtstag. Hrzg. von M. Peckavé. Berlin; New York: Walter de Gruyter (Miscellanea mediaevalia. Bd. 30). S. 581–599.

Suchla B. R., hg. (1990) Corpus Dionysiacum IPseudo-Dionysius AreopagitaDe divinis nominibus. Berlin; New York: Walter de Gruyter (Patristische Texte und Studien. Bd. 33).

Suszko R. (1975) «The Abolition of the Fregean Axiom». Logic Colloquium: Symposium on Logic Held at Boston, 1972–1973. Ed. by R. Parikh. Berlin; New York: Springer (Lecture Notes in Mathematics. Vol. 453). P. 169–239.

Toraldo di Francia G. (1985) «Connotation and Denotation in Microphysics». Recent Developments in Quantum Logics. Ed. by P. Mittelstaedt, E. W. Stachow. Mannheim: Bibliographisches Institut. P. 203–214.

Urquhart A. (2008) «Emil Post». Handbook of the History of Logic. Vol. 5. Logic from Russell to Church. Ed. by D. M. Gabbay, J. Woods. Amsterdam: North Holland – Elsevier. P. 617–666.

Van der Berg R. M. (2008) Proclus’ «Commentary on the Cratylus» in Context. Ancient Theories of Language and Naming. Leiden; Boston: Brill (Philosophia Antiqua. Vol. 112).

Wittgenstein L. (1922) Tractatus logico-philosophicus. German text with an English transl. en regard by C. K. Ogden; with an introduction by B. Russell. London: Routledge and Kegan Paul.

Wittgenstein L. (2015) Tractatus logico-philosophicus. Side-by-side-by-side edition, version 0.42. January 5, 2015. Режим доступа: http://people.umass.edu/klement/tlp/

Wójcicki R. R. (1984) «Suszko’s Situational Semantics». Studia Logica. An International Journal for Symbolic Logic. Vol. 43. № 4. P. 323–340.

Zachhuber J. (2011) «Jean-Luc Marion’s Reading of Dionysius the Areopagite. Hermeneutics and Reception History». Reading Forwards and Reading Backwards. Conversations about Reading the Church Fathers. Ed. by M. Ludlow, S. Douglass. Edinburgh: T&T Clark. P. 3–22.

Баранов В. А. (2007) «К вопросу об экзегезе святоотеческого текста по триадологической проблематике в иконоборческих спорах». «Св. Троица» прп. Андрея Рублева в свете православного апофатизма. 18 ноября 2005 г. Иконоборчество: вчера и сегодня. 22 сентября 2006 г. Материалы конференций. СПб.: Петрополис. C. 127–143.

Бирюков Д. C. (2009) «Тема описания человека через “схождение особенностей” у свт. Василия Великого и ее церковно-исторический и историко-философский контекст». Богословские труды. Вып. 42. C. 87–109.

Васюков В. Л. (2004) «Не-фрегевский путеводитель по гуссерлевским и мейнонговским джунглям. I». Логические исследования. Вып. 11. С. 99–118.

Васюков В. Л. (2005а) «Не-фрегевский путеводитель по гуссерлевским и мейнонговским джунглям. II». Логические исследования. Вып. 12. С. 146–161.

Васюков В. Л. (2005б) Квантовая логика. М.: ПЕР СЭ.

Горбатов В. В. (2011) «О необходимости различения форм и уровней нефрегевости». Логическая семантика: перспективы для философии и языка и эпистемологии. Сборник научных статей, посвященных юбилею Е. Д. Смирновой. Отв. ред.: Е. Г. Драгалина-Черная, Д. В. Зайцев. М.: Креативная экономика. C. 165–182.

Дионисий (2006) Дионисий Ареопагит. Корпус сочинений. С приложением толкований преп. Максима Исповедника. Пер. с греч. и вступ. ст. Г. М. Прохорова. 2-е изд., испр. СПб.: Изд-во Олега Абышко (Библиотека христианской мысли. Источники).

Лурье В. М. (2006) История византийской философии. Формативный период. СПб.: Axiōma.

Лурье В. М. (2016) «Понятие числа в триадологии восточной патристики». ESSE: Философские и теологические исследования. Т. 1. № 1. С. 289–311.

Лурье В. М. (2018) «Модальная онтология Дионисия Ареопагита». ESSE: Философские и теологические исследования. Т. 3. № 1. С. 229–251.


* Статья представляет собой русскую версию статьи «The Philosophy of Dionysius the Areopagite: An Approach to Intensional Semantics», опубликованной в сборнике: Georgian Christian Thought and Its Cultural Context. Memorial Volume for the 125th Anniversary of Shalva Nutsubidze (1888–1969). Ed. by T. Nutsubidze, C. B. Horn, B. Lourié, with the Collaboration of A. Ostrovsky. Leiden; Boston: Brill, 2014 (Texts and Studies in Eastern Christianity. Vol. 2). P. 81–127.

[1] Я предпочитаю называть этого автора так, как он называет сам себя, без приставки «псевдо-». По моему мнению, Corpus Areopagiticum был в основе своей написан Петром Ивиром в 460-е гг. и тогда еще не приписывался Ареопагиту, а нынешняя псевдонимизированная редакция была создана в палестинских монашеских кругах, близких к Петру, вскоре после его смерти в 491 г., около 500 г. См.: Lourié 2010.

[2] Прежде всего, я имею в виду идеи Лейбница об интенсиональности. См.: Lourié 2012.

[3] См. подробнее: Lourié 2013.

[4] В качестве одного из пионерских исследований можно указать на статью: Rhodes 2013; о Дионисии см. на с. 92.

[5] Обычно современные историки параконсистентных логик «перепрыгивают» от античных предшественников своих идей прямо к Николаю Кузанскому, ср.: Priest 1995, 23–24. Относительно фундаментального различия между Дионисием и даже самым близким к нему языческим автором, Проклом, именно в области параконсистентной логики сошлемся на статью К. Стиля (Steel 2003), хотя ее автор не использует термин «параконсистентный» и не обнаруживает знакомства с современной литературой по соответствующим разделам логики. Весьма важны его наблюдения относительно элиминации параконсистентности из рецепции Ареопагита в схоластике, особенно в комментарии на Корпус авторства Альберта Великого, и относительно критики схоластического подхода к Ареопагиту у Николая Кузанского.

[6] Пока что, по моим сведениям, появился только один автор, который рассматривает Дионисия как аналитического философа, — Т. Д. Кнеппер (Knepper 2013). О его взглядах см. ниже, прим. 41. Но уже здесь надо заметить, что у этого автора полностью выпадают из поля зрения параконсистентные аспекты учения Дионисия, и поэтому его Дионисий — это Дионисий латинской схоластики. Ср.: «Dionysian Rule of Reference: God is properly identified as that which cannot be identified by anything of being (i. e., the properties sourced by the divine names themselves), but possesses of everything of being in hyper-being preeminence (i. e., precontains the properties sourced by the divine names themselves)» («Дионисиево правило референции: Бог в собственном смысле слова идентифицируется как тот, кто не может быть идентифицирован посредством чего бы то ни было из бытия (т. е. источником свойств являются сами божественные имена), но обладает всяческим бытием в сверх-бытийном превосходстве (т. е. предсодержит свойства, имеющие источник в самих божественных именах») (ibid., 139); далее формулируется аналогичное «Dionysian Rule of Predication» («Дионисиево правило предикации») (ibid.). Кнеппер не понимает, что у «несхоластического» Ареопагита всякое hyper-being имеет также смысл «вообще-не-бытия», и поэтому никакой вариант ансельмова онтологического аргумента не будет приложим к Богу Дионисия. Подробнее Кнеппер развивает свое понимание апофатизма Дионисия в статье «Не не: метод и логика Дионисиева отрицания» (Knepper 2008), основная мысль которой такова: «It is not the case that Dionysius’s negation of predicate terms should be read propositionally, that is to say, as It is not the case that God is p. Rather, when interpreted apophatically, Dionysius’s notp signifies morepthanmostp» («Речь идет не о том, что отрицание Дионисием предикативных терминов следует читать пропозиционально, иначе говоря, как: Не имеет место такое, что Бог есть р. Напротив апофатическая интерпретация предполагает, что Дионисиево не-р означает более-p-чем-наибольшее-р»). Это и есть то, что я выше назвал элиминацией параконсистентности. В противном случае, т. е. при учете параконсистентности богословия Дионисия, следовало сказать, что пропозиции God is p и God is notp взятые совместно являются актуально истинными.

[7] Цитирую по электронному изд.: Wittgenstein 2015, основанному на изд.: Wittgenstein 1922.

[8] Таким образом, я поддерживаю Карлоса Стиля в его неприятии «the later speculative dialectic of German idealism» («позднейшей спекулятивной диалектики немецкого идеализма») (Steel 2003, 599), который черпал свое вдохновение из Кузанца и, следовательно, также, косвенно, из Дионисия, но не разделяя реальной антиспекулятивной философской программы Николая Кузанского.

[9] В качестве одной из первых попыток подобного исследования см. рассуждение Р. Мортли: Mortley 1986, 221–241 (ch. XII. Pseudo-Dionysius: a positive view of language and the via negativa), — но автор целиком погружен в неоплатонистический контекст и почти не учитывает контекст патристики.

[10] Как свежий пример такого подхода можно порекомендовать книгу: Schäfer 2006.

[11] Относительно неоплатонистического фона этой концепции Дионисия, особенно прокловой интерпретации фрагмента 142a3–6 из платоновского «Парменида», см.: Van der Berg 2008, 216–217. См. также: Corsini 1962, 104 и везде.

[12] Тексты Corpus Areopagiticum будут цитироваться по критическим изданиям: Suchla 1990 (для «De divinis nominibus» = DN) и Heil, Ritter 1991 (для «De mystica theologia» = MT и Epistolae), но ссылки в тексте будут даваться следующим образом: сокращенное название (DN, MT, Epistolae), глава, параграф, пагинация страниц и строк критического издания и, после косой черты (/) пагинация столбцов и их частей (от A до D) в PG 3. Рус. пер. Г. М. Прохорова по изд.: Дионисий 2006, — но с моими изменениями, которые специально оговариваться не будут.

[13] «Таким образом, всеобщая, все превышающая Причина гармонизируется и с неименуемостью, и со всеми именами сущего <…> Ибо она — Причина не только связи, жизни или совершенства, чтобы всего лишь от одного или другого из этих попечений называться сверхименной благостью: все сущее она заранее просто и неограниченно содержала в себе все приводящими в исполнение благостями единого своего всепричинного Промысла, и всеми существами гармонично воспевается и именуется» (ср.: Дионисий 2006, 138).

[14] «…богословы воспевают ее [причину всего сущего] и как неименуемую, и от [или: посредством] всякого имени» (ср.: Дионисий 2006, 136).

[15] «Слово осмелится даже сказать, что и не сущее причастно Прекрасному-и-Благому, ибо и оно — прекрасно и благо тогда, когда, по отъятии всего, сверхсущественно воспевается в Боге» (ср.: Дионисий 2006, 179–180).

[16] См. подробнее: Lourié 2013.

[17] Термин из натурфилософии стоиков. Буквально означает «совместное дыхание», что подразумевает внутреннюю гармонию, в которой находятся все творения друг с другом.

[18] «И существует также божественнейшее познание Бога, осуществляемое через незнание путем превосходящего ум единения, когда ум, отступив от всего сущего, оставив затем и самого себя, соединяется с пресветлыми лучами, оттуда и там освещается недоступной исследованию глубиной Премудрости (ср.: Прем. 8:1). Хотя, как я сказал, подобает ее познавать и во всем, ибо она, согласно Речениям, создательница всего (ср.: Прем. 7:21, 8:30; Пс. 103:24), всегда всё гармонизирующая, и причина неразрушаемых гармонизации и порядка всего, постоянно соединяющая завершения первых с началом вторых, добротворящая [или: прекрасно творящая] единое совоздыхание и гармонию всего» (ср.: Дионисий 2006, 257).

[19] О понятиях διακρίσεις и πρόοδοι в Боге см., прежде всего, классическую работу: Lossky 1930 (хотя есть и русский перевод этой работы, как в этом, так и в других случаях я не буду приводить точную библиографию имеющихся русских переводов и ссылаться на них). См. также: Golitzin 1994, 54–61.

[20] «Эти общие и соединенные разделения или благолепные исхождения всецелой божественности мы постараемся по мере сил воспеть, руководствуясь богоименованиями (божественными именами), которые являют ее в Речениях, — прежде, как уже сказано, отдав себе отчет в том, что все благотворящие богоименования, применяемые к богоначальным ипостасям, следует воспринимать как относящиеся ко всей богоначальной целостности без изъятия» (ср.: Дионисий 2006, 160).

[21] То есть понимая интенсиональность приблизительно в смысле фрегевского Sinn. Ср. подобный подход в новейшей литературе: Chalmers 2002. Ср. также: Church 1951.

[22] См. особенно: Shani 2007.

[23] Относительно «интенсиональных сущностей» и различных попыток их редукции см. весьма содержательную энциклопедическую статью: Bealer 2000. Обычно термин «intensional entities» переводят на русский как «интенсиональные сущности», но для работ в области патристики такой подход недопустим, так как слово «сущность» также является термином, причем таким, который ни в коем случае не должен применяться к энергиям.

[24] Подход, строго противоположный подходу Крипке, — «модальный реализм» Дэвида Льюиса, в котором все возможные миры считаются реально существующими (Lewis 1986).

[25] Так Рене Рок обозначил всю интеллектуальную конструкцию Дионисия, см.: Roques 1983.

[26] Ср. также: «ἡ ὑπέρθεος θεότης» («сверхбожественная божественность», DN 2.4, 126.15–16/641A) и «ὑπέρθεος ὑπερουσίως εἷς θεός» («сверхбожественный сверхсущностный единый Бог», DN 2.11, 136.15–16/649C). Относительно функции превосходной степени в языке Дионисия см.: Scazzoso 1967, 35–46.

[27] Выражаю свою благодарность Виктору Горбатову, который привлек мое внимание к двумерной семантике.

[28] Это общее место восточнохристианской мистической традиции. У Дионисия см., прежде всего, то, что он пишет в «О божественных именах» про своего учителя Иерофея Афинского, который «οὐ μόνον μαθὼν ἀλλὰ καὶ παθὼν τὰ θεῖα» («не только изучал, но и претерпевал божественное», DN 2.9, 134.1–2/648B), и MT всю целиком. Ср.: Golitzin 2007, особ. 162–165.

[29] Безотносительно к деталям логической интерпретации, которые тут могут быть разными, ясно, что Дионисиевы ὑπερ- выступают в качестве логических операторов, определенным образом трансформирующих объекты тварного мира, к которым их применяют.

[30] Здесь мы вынуждены уклониться от подробного обсуждения — весьма интересной самой по себе — проблемы денотатов отсутствующих (отрицаемых) объектов. Подобные логические объекты, включая μὴ ὄν, суть онтологические дырки в творении, а поэтому, в качестве дырок, имеют денотаты. Свой денотат у бублика, но свой — и у дырки от бублика.

[31] См., однако, ниже (раздел 8) относительно квази-предикативного подхода.

[32] Об именах собственных как жестких десигнаторах см.: Kripke 1980, 18–19 и 233–234. Крипке возвращается здесь к подходу Джона Стюарта Милля, отброшенному было Фреге и Расселом; см.: Mill 2001 (из его книги «System of Logic», book I, ch. 2, sect. 5, впервые опубликованной в 1881 г.). Тут может возникнуть вопрос, почему я предпочитаю здесь обсуждать понятие жесткой десигнации Крипке, а не прямой десигнации Рут Баркан Маркус и Дэвида Каплана. Действительно, если бы мы рассматривали сочинения Ареопагита вырванными из того контекста патристической традиции, в котором они написаны, — а это в первую очередь творения Каппадокийцев, — то мы не имели бы оснований различить у него понятия жесткого десигнатора как предельного случая дескрипции (Крипке) и имени собственного как простого «tag» («метки») — чистого Bedeutung без всякого Sinn (ср.: Barcan Marcus 1993). Однако Дионисий писал внутри патристической традиции, которая была явным образом дескрипционистской. Мы остановимся на этом подробнее в заключительных разделах статьи, когда будем говорить о Василии Великом. Впрочем, будем ли мы сравнивать божественные имена с жесткими десигнаторами или с «метками», мы придем к одним и тем же содержательным выводам, хотя формально наши рассуждения будут слегка различаться.

[33] Но, между прочим, подобная применимость теории референции Крипке к богословской антропологии говорит скорее о ее слабости в качестве общей теории референции. Дело в том, что в богословской антропологии главная проблема референции уже a priori решена, но самым грубым «haecceit-ическим» (от термина Дунса Скота haecceitas «этость») путем: различные индивидуумы всегда, во всех возможных мирах, уже раз и навсегда безошибочно индивидуализированы без всякой логической процедуры — просто с точки зрения Творца. См. особо релевантную для нас критику теории референции Крипке: Hintikka, Sandu 1995 (переиздание: Hintikka 1998, 175–218).

[34] Переиздание: Stead 2000, ch. XX (в издании отсутствует сквозная нумерация страниц).

[35] Подробнее см.: Lourié 2010.

[36] О нем см. теперь: Peltomaa 2001.

[37] Ср.: Leezenberg 2001, где автор дает обзор истории вопроса о природе метафоры от Аристотеля до, примерно, 1980-х гг. (но не упоминая подхода Хинтикки и Санду, о котором см. ниже). [Добавление 2018 года: см. теперь подробное обсуждение проблемы построения формальной семантики непрямых значений в моих совместных с О. В. Митрениной работах: Lourié, Mitrenina 2015; Lourié, Mitrenina 2019 (в печати).]

[38] В том смысле, что метафору невозможно заменить дескрипцией, не потеряв при этом того самого компонента значения, ради которого была выбрана метафора, а не другой способ изложения мысли.

[39] Критику дэвидсоновского подхода к метафоре см., в частности, в статье: Kittay 1995. Дискуссия продолжается. Недавнюю защиту подхода Дэвидсона см., например, в статье: Lepore, Stone 2010.

[40] Переиздание: Hintikka 1998, 274–310. Было справедливо замечено, что данный подход, с одной стороны, действительно игнорирует психологию, но, с другой стороны, он и не создает никаких препятствий для изучения метафоры в рамках когнитивной семантики, ср.: Engström 2001. Тем не менее подход к метафорам в когнитивной семантике (Лакофф, Джонсон и др.) вряд ли может считаться особенно перспективным для изучения ее логической структуры, ср.: Pietarinen 2008. Т. Д. Кнеппер предлагает другой подход к семантике Ареопагита, основываясь на теории речевых актов Джона Сёрля и теории метафоры Джорджа Лакоффа и Марка Джонсона (Knepper 2013). Его наблюдения мне представляются полезными для изучения Ареопагита, но скорее в области когнитивной психологии, нежели семантики. Что касается подхода Сёрля к проблеме непрямых значений (включая поэтические тропы), то он мне не кажется правильным; я разделяю в особенности ту критику этого подхода, которая была высказана Дэвидсоном, а также Хинтиккой и Санду. Кнеппер использует теорию речевых актов некритически и, самое главное, не упоминает о трактовке Сёрлем метафоры в свете этой его теории.

[41] См.: Jakobson 1971, особ. 254–259 (раздел «The Metaphoric and Metonymic Poles»).

[42] Идея межмировых линий была впервые сформулирована для многомировой семантики Дэвидом Капланом в статье 1967 г. «Межмировые линии», которая интенсивно обсуждалась несколько лет, не будучи опубликованной, причем сам Каплан успел отказаться от этой своей идеи. Но прошло еще несколько лет, и статью, которая уже заняла видное место в истории обсуждения темы, все-таки издали, а Каплану пришлось сделать к ней примечание о том, что он давно уже с ней не согласен: Kaplan 1979 (ср. сноску на с. 88). Таким образом, Каплан присоединился к большинству философов того времени, которые считали проблему межмировой идентичности псевдопроблемой. Тем не менее Яаакко Хинтикка воспринимает эту проблему весьма серьезно. Ср., в частности: Hintikka 1974, а также прим. 49 ниже.

[43] Выражаю большую признательность Виктору Горбатову, который познакомил меня с этими и другими трудами В. Л. Васюкова. Васюков, в свою очередь, опирается на работы Романа Сушко по нефрегевской семантике; см. о них в целом: Wójcicki 1984.

[44] Ср.: «[Бог] — причина всякого бытия, но Он же и ничто (μὴ ὄν), так как превыше всякой сущности» («αἴτιον μὲν τοῦ εἶναι πᾶσιν, αὐτὸ δὲ μὴ ὂν ὡς πάσης οὐσίας ἐπέκεινα») (DN 1.1; 109.15-16/588B) (ср.: Дионисий 2006, 124); «это (Бог) всего сущего причина, но само оно ничто (οὐδὲν), потому что (выше) всего пресущественно изъято» («ὅτι πάντων μέν ἐστι τῶν ὄντων αἴτιον, αὐτὸ δὲ οὐδὲν ὡς πάντων ὑπερουσίως ἐξῃρημένον») (DN 1.5; 117.3–4/593C) (ср.: Дионисий 2006, 134).

[45] Ср. выше, прим. 7.

[46] См. подробно: Lourié 2013 (рус. версия: Лурье 2018). Полная система аксиом этой онтологии NR + K + D + CD, а итеративные аксиомы (S4, S5, B) не выполняются.

[47] Подробное обсуждение см., прежде всего, в статье: Parsons 1967; cр.: Catterson 2004.

[48] Очевидно, что то же самое приложимо к метонимии.

[49] Многомировое обобщение фрегевской семантики было предвосхищено Карнапом и впервые предложено в подробно разработанном виде Ричардом Монтегю; см.: Montague 1974, везде, но особ. 119–147 («Pragmatics and Intensional Logic» (1970), где особ. 127–131. Об эквивалентности интенсионалов Монтегю межмировым линиям Хинтикки см.: Hintikka 1989.

[50]+ — множество натуральных чисел (ℕ) за исключением 0.

[51] Или, согласно альтернативной формулировке того же самого вывода Фреге Дэвидсоном, все истинные предложения имеют референцией один и тот же «Великий Факт», который представляет собой не что иное, как состояние дел во всем мире вообще (Davidson 1991a).

[52] Особенно вслед за Джоном Барвайзом и Джоном Перри, которые опубликовали самую первую редакцию своих взглядов в статье: Barwise, Perry 1981. О дальнейшем развитии их взглядов и их разногласиях см., например: Barwise 1989, 242–251.

[53] См. особенно: Quine 1969.

[54] Чтобы переписать формализм Патнема для референции через метафору или метонимию, нужно будет использовать вместо интенсионалов Монтегю смысловые линии, а вместо самих индивидуумов их свойства. Чтобы переписать тот же формализм для божественных имен, нужно рассматривать вместо произвольного предиката Fi, который при интерпретации I имеет в возможном мире Wj экстенсионалы Rij, предикат «быть Богом», который имеет только один индивидуум в качестве экстенсионала, причем он один и тот же для всех возможных миров.

[55] Ср., относительно возможных миров художественной литературы: Doležel 1998, и специально относительно поэзии: Semino 1997.

[56] Точнее, мы говорим тут лишь о тех логиках, в которых не делается попыток избежать коллапса волновой функции, и где, кроме того, Копенгагенская интерпретация квантовой теории воспринимается весьма серьезно. Ср. современную классификацию квантовых логик, предложенную не так давно Хилари Патнемом: Putnam 2005. В этой самой недавней своей публикации по квантовой логике Патнем, чьи взгляды достаточно серьезно менялись со времени его первых публикаций по квантовой логике в начале 1960-х гг., остается совершенно непреклонным в своем неприятии «Bohr’s rejection of scientific realism» («боровского отказа от научного реализма») (Putnam 2005, 625 (прим. 13)) и выражает свое крайнее разочарование неклассическими квантовыми логиками. Тем не менее неклассические параконсистентные логики все равно рассматриваются как потенциально полезные для будущего развития квантовых логик: Cattaneo, Dalla Chiara, Giuntini, Paoli 2009, особ. 173–175 (логическое представление Боровской дополнительности). Понятно, что Дионисий Ареопагит еще менее, чем Бор, удовлетворил бы критерию научного реализма Патнема. Его катафатический и апофатический подходы — лишь один из частных случаев дополнительности, которая, по «нереалистическому» мнению Бора, фундаментальна для человеческого познания как такового. Cp. в связи со сказанным: Bohr 1958.

[57] См. подробнее: Krause, da Costa 2006; Krause, da Costa 2004.

[58] Референт частично дескриптивного имени определен отчасти обладанием некоего дескриптивного свойства, а «…in part by the same nondescriptive mechanisms that determine the reference of ordinary nondescriptive names» («…отчасти теми же недескриптивными механизмами, которые определяют референцию обычных недескриптивных имен» (Soames 2002, 51).

[59] См. некоторые недавние работы по формальной семантике метафор: Mao, Zhou 2007 (на основе теории Лакоффа и Джонсона) и Pietarinen 2008 (на основе теории Хинтикки и Санду, но с использованием методов Ч. Пирса).

[60] Семантика, основанная на смысловых линиях (хотя и без употребления этого термина, и без ссылок на Хинтикку и Санду), уже фактически разработана Владимиром Васюковым для других целей (см. выше, прим. 44). Он назвал свою разработку «метафорической логикой» и выстроил ее как нефрегевскую и несушковскую. Основная идея заключается в дальнейшем ослаблении нефрегевской версии так называемого «принципа идентичности» Лейбница (я буду продолжать держаться традиционного названия этого принципа, хотя сам Лейбниц в последние месяцы жизни, в 1716 г., в пятом письме к Кларку, успел от него отказаться, см.: Chernoff 1981). Этот принцип может быть следующим образом сформулирован в нефрегевской ситуационной семантике: (ab) ↔ ∀φ(φ(a) ⇒ φ(b)), где φ — некая формула, ab означает «a ситуационно влечет за собой b», ⇒ нефрегевский коннектив «референциально ведет к» (так что ⇔ будет идентичен коннективу Сушко ≡ «(экстенсионально/референциально) идентично»). Эквивалент принципа Лейбница в метафорической логике — принцип подобия неразличимых с предвзятой точки зрения: (ab) ↔ ∃φ(φ(a) φ(b)), что означает то, что, по крайней мере, одна ситуация, где встречается a, должна иметь некоторое отношение (с той самой предвзятой точки зрения) к ситуациям, где встречается b. Очевидно, что последний принцип слишком слабый, чтобы быть приложимым к божественным именам, но зато он, по всей видимости, очень подходит для формализации теории Хинтикки и Санду.

[61] В качестве пионерских работ надо назвать: Goldblatt 1974 и Toraldo di Francia 1985. Васюков отмечает, что Гольдблатт использовал в своих фреймах Крипке функции, аналогичные интенсионалам Монтегю (Васюков 2005б, 105–106).

[62] См., прежде всего, подробное обсуждение проблем десигнации и дескрипции (включая невозможность «жесткой десигнации» в смысле Крипке) в статье: Dalla Chiara, Toraldo di Francia 1992. См. также другие важные работы: Dalla Chiara 1987; Dalla Chiara, Giuntini, Krause 1998; Dalla Chiara, Giuntini, Greechie 2004, особ. 199–200). См. также следующее примечание.

[63] В начале 1950-х гг. Эрвин Шрёдингер в своих популярных лекциях о квантовой механике сравнил неразличимость квантовых объектов с неразличимостью долларов на банковском счете; к началу 1960-х гг. этот пример приобрел огромную популярность; об истории этого сравнения см.: French, Krause 2006, 142–143, 159, 220, 370–371.

[64] Ср. краткое, но очень полезное введение к этой проблематике у Дионисия в схолии начала VI в. к «О небесной иерархии» 15.4, к тому месту, где Дионисий цитирует Еф. 3:10 о «многоразличной премудрости Божией» («πολυποίκιλος σοφία τοῦ θεοῦ») (PG 4, 109AB).

[65] Наши сегодняшние представления об этой полемике суммируют монографии: Drecoll 1996 и Hildebrand 2007.

[66] Остается особенно важной статья: Krivochéine 1985, хотя автор не упоминает Дионисия ни единым словом. В более широкой перспективе см.: Larchet 2010, где значительная часть книги посвящена Каппадокийцам.

[67] Вопрос об обоснованности и ограничениях приложимости Василиевой теории собственных имен к Богу даже не ставится ни в одной из трех основных работ в этой области: Kalligas 2002; Robertson 2002 (статья ограничена Василием, остальные отцы даже не упоминаются); Бирюков 2009. Благодарю Дмитрия Бирюкова за обсуждение со мной этих тем. Недавняя монография М. Дель Кольяно (Del Cogliano 2010) в основном следует за Каллигасом и Робертсоном в понимании Василиевой теории собственных имен, но добавляет некоторые подробности относительно философского бэкграунда (ibid., 191–196).

[68] См., прежде всего: Robertson 2001, особ. 279–280, и вслед за ним: Radde-Gallwitz 2009, 114–122 (раздел «A Central Tension: ‘Common Usage’ and Purification»). Эти страницы создают контекст, необходимый для правильного восприятия выводов тех работ, которые перечислены в предыдущем примечании.

[69] О понятии «характúр» в богословии иконы см.: Баранов 2007, а для более позднего периода — Lourié 2006.

[70] См. особенно его статью 1975 г.: Putnam 1979.

[71] См.: Searle 1958, особ. 172–173. В более широкой перспективе см.: Searle 1983, 231–261 (ch. 9. Proper Names and Intentionality).

[72] Как выражается сам Василий (Contra Eunomium I.5), «…τῆς Πατρὸς φωνῆς ἴσον δυναμένης τῷ ἀγεννήτῳ…» («…поскольку термин “Отец” имеет эквивалентное значение термину “нерожденный”…») (Sesboüé 1982, 176). Подробнее см.: Robertson 2001, 277–278 и везде.

[73] Contra Eunomium II.10.25–27: «Οὐ γὰρ ἐν τῷ πρὸς τί πως ἔχειν ἡ διαφορὰ τῷ Υἱῷ πρὸς τὰ ἄλλα, ἀλλ’ ἐντῇ ἰδιότητι τῆς οὐσίας ἡ ὑπεροχὴτοῦ θεοῦ πρὸς τὰ θνητὰ διαφαίνεται» (Sesboüé 1983, 40), цит. у A. Радде-Гальвиц (Radde-Gallwitz 2009, 116 (прим. 9)).

[74] Кроме имени «Сын». Бог именуется Отцом вне триадологического контекста (например, в молитве «Отче наш»), однако именование Cыном вне триадологического (или подобного ему) контекста вообразить невозможно. Но эта особенность имени Сын имеет богословскую, а не логическую природу.

[75] «Также и то, что Отец — источная божественность, а Сын и Дух — богородящей божественности, если можно так выразиться, прозябения богорожденные и как бы цветы и пресущественные светы, как мы восприяли из священных Речений» (ср.: Дионисий 2006, 153).

[76] Тернарное эксклюзивное «или» было впервые описано в пионерской работе польского логика Эмиля Поста о логических коннективах (1941), которая была спасена от забвения только в 1990-х гг. См.: Urquhart 2008.

Закладка Постоянная ссылка.

Комментарии запрещены.